You are on page 1of 56

Question Paper [CODE - 12599]

NEET PATTERN TEST Brahmastra Major Test-03


13th NEET - Phase 11
KOTA

Date: 19-Mar-2023 Duration: 3 Hours 20 Mins Max Marks: 720


SYLLABUS

Biology

The Living World,Biological Classification,Plant Kingdom,Strategies for Enhancement in Food Production


(plant Breeding),Reproduction : Reproduction in Organism,Sexual Reproduction in Flowering
plants,Morphology of Flowering Plants,Anatomy of flowering Plants,Transport in Plants,Photosynthesis
in Higher Plants,Mineral Nutrition,Respiration in Plants,Plant Growth and Development,Organisms and
populations,Ecosystem,Biodiversity and Conservation,Environment Issues,Genetics: Principle of
Inheritance and Variation,Molecular Basis of Inheritance,Biotechnology Principles and
Processes,Biotechnology and its Applications,Human Reproduction,Reproductive Health,Animal
Kingdom,Human Health and disease,Structural Organization in Animals,Neural Control and
Coordination,Breathing And Exchange Of Gases (Respiratory System),Excretory Products and their
Elimination,Locomotion and Movement,Chemical Coordination and Integration,Digestion an
Absorption,Cell The Unit of Life,Cell Cycle and Cell Division,Evolution : Origin And Evolution,Strategies
for Enhancement in Food Production (Animal Husbandry),Biomolecule,Body Fluids And Circulation
(Circulatory System),Microbes in Human Welfare

Physics

Physical World and Measurement,Kinematics,Circular Motion,Laws of Motion and Friction,Work,Energy


and Power,Motion of System of Particles and Rigid Body,Ray Optics and Optical Instruments,Wave
Optics,Electromagnetic Induction,Alternating Currents,Magnetic Effects of Current and
Magnetism,Electromagnetic Waves,Gravitation,Oscillations and Waves,Properties of Bulk
Matter,Electrostatics,Current Electricity,Capacitance,Thermodynamics,Behaviour of Perfect Gases and
Kinetic Theory of Gases,Dual Nature of Radiation and Matter,Atoms,Nuclei,Semiconductor -Electronics:
Materials,Devices and Simple Circuits

Chemistry

Some Basic Concepts of Chemistry,Structure of Atom,Redox Reactions,Chemical Equilibrium,Ionic


Equilibrium,Thermodynamics & thermochemistry,States of Matter: Gases and Liquids,Solid
State,Solutions,Electrochemistry,Chemical Kinetics,Surface Chemistry,Classification of Elements and
Periodicity in Properties,Chemical Bonding and Molecular Structure,Hydrogen,s -Block Elements,Some p
-Block Elements,p-Block Elements,d-and f-Block Elements,Coordination Compounds,Metallurgy,Organic
Chemistry: Some basic Principles and Techniques,Hydrocarbons,Aldehydes,Ketones and Carboxylic
Acids,Amines,Haloalkanes and Haloarenes,Alcohols,Phenols and Ethers,Biomolecules
Biology

1. During favourable condition the encysted 1. अनुकु ल परिस्थितियो में एनसिस्ट युक्त अमीबा
amoeba divides by multiple fission and बहुविखण्डन द्वारा विभाजित होकर स्युडोपोडियोस्पोर
produces pseudopodiospores. The उत्पन्न करता है इस घटना को क्या कहते है 
phenomenon is known as:
(A) मुकु लन 
(A) Budding
(B) बीजाणु जनन 
(B) Sporulation
(C) Fragmentation (C) विखण्डन 

(D) Regeneration (D) पुनरूद्भवन

2. ATP molecules required for synthesis of 2. C4 पथ द्वारा ग्लूकोज के एक अणु के संश्लेषण के लिए
one molecules glucose by C4 pathway are- कितने ATP अणु  की आवश्यकता है-
(A) 12 (A) 12
(B) 18 (B) 18
(C) 24 (C) 24
(D) 30 (D) 30
3. A group of closely related, structurally and 3. निकट सम्बन्धित, संरचनात्मक एवं कार्यात्मक समान अंगों
functionally similar organisms is known as के जन्तुओं के समूह को क्या कहते हैं
(A) Genus (A) वंश
(B) Species
(B) जाति
(C) Sibling species
(C) सिबलिंग स्पीशीज
(D) None of them
(D) इनमें से कोई भी नहीं
4. Mendel's principle of segregation means 4. मेंडल के पृथकरण के सिद्धान्त का अर्थ है कि जनन
that the germ cells always receive कोशिकाएँ हमेशा प्राप्त करती है
(A) One pair of alleles (A) एलील का एक युग्म
(B) One quarter of the genes (B) जीनो का एक चौथाई
(C)  One of the paired alleles
(C) युग्मीन एलील्स में से एक 
(D)  Any pair of alleles
(D) एल्लील का कोई भी युग्म

5. Which of the following statements 5. निम्नलिखित में से कौनसे कथन सही है ?


are  correct ? (i) दीर्घीकरण क्षेत्र से कु छ अधिचर्मीय
कोशिकाएँ मूल
(i) From the region of elongation, some of रोम बनाती है। 
the epidermal cells form root hairs.
(ii) राइजोफोरा
में श्वसन मूल देखी जाती है।
(ii) Pneumatophores are seen in
(iii) बरगद के पेड में अपस्थानिक मूल देखी
जाती है। 
Rhizophora.

(iii) Adventitious roots are seen in the (iv) मक्का तथा गन्ना में प्रोप मूल (सहारा देने वाली मूल)
banyan tree. होती है। 
(iv) Maize and sugarcane have prop roots.
(A) (i) तथा (iv)
(A) (i) and (iv)
(B) (i), (iii) तथा (iv)
(B) (i), (iii) and (iv)
(C) (iii) तथा (iv)
(C) (iii) and (iv)
(D) (ii) तथा(iii)
(D) (ii) and (iii)
6. Addition of which component into a 6. एक पारितंत्र बनाने वाले समुदाय मे कौनसे घटक जोड़े
community makes it ecosystem - जाते हैं -
(A) Microbes (A) सुक्ष्मजीव
(B) Abiotic factors
(B) अजैविक कारक
(C) Humus
(C) ह्यूमस
(D) Plants
(D) पादप
7. Match column-I, II and III and choose the 7. कॉलम-I, II तथा III का मिलान करे तथा सही विकल्प
correct option. का चयन करे -
  Column-I   Column-II   Column-III   कॉलम-I   कॉलम-II   कॉलम-III

(A) Marginal I. p. Sunflower, Marigold


(A) सीमान्त I. p. सूरजमुखी, गेंदा

(B) Axile II. q. Dianthus, Primrose (B) अक्षीय II. q. डायऐंथस, प्रिमरोज

(C) Parietal III. r. Mustard, Argemone


(C) भित्तीय III. r. सरसो, आर्जीमोन

Free China rose, Tomato,


(D) IV. s. (D) मुक्त के न्द्रकी IV. s. गुड़हल, टमाटर, नींबू
central Lemon

(E) Basal V. t. Pea (E) अधारी V. t. मटर

(A) A-V, t; B-II, s; C-I, r; D-III, q; E-IV,p (A) A-V, t; B-II, s; C-I, r; D-III, q; E-IV,p
(B) A-I, t; B-II, s; C-III, r; D-IV, p; E-V,q (B) A-I, t; B-II, s; C-III, r; D-IV, p; E-V,q
(C) A-V, p; B-II, s; C-I, q; D-III, r; E-IV,t (C) A-V, p; B-II, s; C-I, q; D-III, r; E-IV,t
(D) A-V, p; B-III, q; C-II, s; D-I, t; E-IV,r (D) A-V, p; B-III, q; C-II, s; D-I, t; E-IV,r
8. Filiform apparatus are located at the :- 8. तंतुमय समुच्चय (apparatus) कहाँ उपस्थित होते हैं :-
(A) Tip of microsporangium (A) लघु बीजाणुधानी के शीर्ष पर
(B) Tip of pollen tube (B) परागनलिका के शीर्ष पर
(C) Tip of megasporangium
(C) गुरूबीजाणुधानी के शीर्ष पर
(D) Base of megasporangium
(D) गुरूबीजाणुधानी के आधार पर
9. Which statements about photosynthesis 9. प्रकाश संश्लेषण के बारे में कौन से कथन सही हैं?
are correct?
(I) C4 चक्र में प्रथम CO2 ग्राही PGA है
(I) First CO2 acceptor in C4 Cycle is PGA
(II) C3 पौधों में, प्रकाश संश्लेषण का प्रथम स्थिर
(II) In C3 plants, first stable product of उत्पाद RuBP है
photosynthesis is RuBP
(III) चक्रीय फोटोफॉस्फोराइलेशन के परिणामस्वरूप
(III) Cyclic photophosphorylation results in
formation of ATP ATP का निर्माण होता है

(IV) Oxygen liberated during (IV) प्रकाश संश्लेषण के दौरान मुक्त हुई ऑक्सीजन
photosynthesis comes from water जल से प्राप्त होती है

(A) I and II alone are correct (A) I और II सही हैं

(B) I and III alone are correct (B) I और III सही हैं
(C) III and IV alone are correct (C) III और IV सही हैं
(D) II and III alone are correct (D) II और III सही हैं
10. Which of the following statements about 10. निम्न में से कौनसा कथन वर्गीकरण के लिए सत्य नही है
classification is not true- I.एक कु ल के सदस्य एक जीनस मे सम्मिलित सदस्यो
I.Members of a family are less than similar की तुलना में समान है
than members of an included genus
II.एक गण में एक वंश में सम्मिलित सदस्यो की संख्या
II.An order has more members than the
से अधिक सदस्य होते है
number of members in an included genus
III.Families have more members than III.संघ की अपेक्षा कु ल में अधिक सदस्य होते है
phyla IV. वर्गक में जातियों की संख्या उनसे संबंधित समानता
IV.The number of species in a taxon के परिमाण पर निर्भर करती है.
depends on  their relative degree of
(A) के वल III 
similiarity.
(A) Only III (B) के वल IV

(B) Only IV (C) के वल II

(C) Only II (D) कोई नही

(D) None
11. A gene producing multiple effect is called - 11. एक जीन जो बहुगुणिता के प्रभाव को उत्पन्न करती  है,
(A) Pleiotropic gene वह कहलाती है -

(B) Split gene (A) पीलियोट्रोपिक जीन    

(C) Complementary gene (B)  विपाटित जीन

(D) Supplementary gene (C) सम्पूरक जीन

(D) पूरक जीन

12. Which zone is not found in pond: 12. कौनसा क्षेत्र तालाब में नही पाया जाता है -
(A) Euphotic (A) सुप्रकाशित
(B) Disphotic (B) मंदप्रकाशित
(C) Aphotic (C) अप्रकाशित
(D) All of these
(D) ये सभी
13. Identify shape of bacteria 13. जीवाणु के आकार को पहचाने

(A) a = cocci, b = rod – shaped, c = (A) a = कोकई, b = छड़ाकार, c = बैसिलाई, d = 


bacilli, d = comma – shaped कॉमा-आकार
(B) a = spherical coccus, B = Bacilli, c =
(B) a = गोलाकार कोकस, b = बैसिलाई, c =
spirilla, d = vibrio
सर्पिलाकार, d= विब्रियो
(C) a = cocci, b = spirilla, c = vibrio, d =
Bacilli (C) a = कोकई, b = सर्पिलाकार, c = विब्रियो, d
= बैसिलाई 
(D)  a = vibrio, b = spirilla, c = bacilli, d =
coccus (D) a = विब्रियो, b = सर्पिलाकार, c = बैसिलाई, d
=  कोकस
14. In the given figure of phloem tissue, 14. दिये गये उतक मे चिन्हित (A,B,C)  मे से पहचान कर
identify the marked part (A, B and C) बताइए की कौन चालनी नलिका मे दाब प्रवणता को
which help in maintaining the pressure नियंत्रित करता है-
gradient in the sieve tubes-

(A) A (A) A

(B) B (B) B

(C) C (C) C

(D) None of the above (D) इनमे से कोई नही

15. Match the followings and choose correct 15. निम्नलिखित को सूमेलित करे तथा सही विकल्प का चयन
option करें -
  Column-I   Column-II   कॉलम -I   कॉलम -II
(a) Aleurone layer (p) without fertilization
(a) ऐल्यूरॉन परत (p) निषेचन रहित 
(b) Parthenocar pic (q) Nutrition fruit
(b) अनिषेेकफलन  (q) पोषण फल
(c) Ovule (r) Double fertilization
(c) बीजाण्ड (r) दोहरा निषेचन 
(d) Endosperm (s) Seed
(d) भ्रूणपोष (s) बीज 
(A) a-p, b-q, c-r, d-s
(A) a-p, b-q, c-r, d-s
(B) a-q, b-p, c-s, d-r
(B) a-q, b-p, c-s, d-r
(C) a-s, b-q, c-p, d-r
(C) a-s, b-q, c-p, d-r
(D) a-q, b-s, c-p, d-r
(D) a-q, b-s, c-p, d-r
16. Given below are some reactions and 16. नीचे कु छ अभिक्रियाएं तथा सम्मिलित एं जाइम दिए गए
enzymes involved. Identify the correct हैं। सही युग्मों को पहचानिए-
pairs-
फ्रु क्टोज-1,6 डाइफॉस्फे ट→
Fructose-1,6 (a) (i) एनोलेज
3PGAL + DHAP
(a) diphosphate → 3PGAL (i) Enolase
+ DHAP (b) सिट्रेट → सिस-एकोनिटेज (ii) थायोकाइनेज
(b) Citrate → Cis-aconitase (ii) Thiokinase (c) सक्सिनिल Co-A → सक्सिनेट (iii) एकोनाइटेस
Succinyl Co-A →
(d) 2PGA ​PEP (iv) एल्डोलेस
(c) (iii) Aconitase →
Succinate
(d) 2PGA → PEP (iv) Aldolase (A) a-iv, b-iii, c-ii, d-i
(B) a-i, b-ii, c-iii, d-iv
(A) a-iv, b-iii, c-ii, d-i
(C) a-ii, b-i, c-iv, d-iii
(B) a-i, b-ii, c-iii, d-iv
(D) a-iii, b-iv, c-i, d-ii
(C) a-ii, b-i, c-iv, d-iii
(D) a-iii, b-iv, c-i, d-ii
17. In a random mating population frequency 17. एक यादृच्छिक मैथुन समष्टि में अप्रभावी जीनोटाइप की
of recessive genotype is 9% then what is आवृत्ति 9% है तो समजात प्रभावी जीनोटाइप व
the frequency of homozygous dominant विषमजात जीनोटाइप की आवृत्ति क्या है
genotype and heterozygous genotype:
(A) 1% तथा 18%
(A) 1% and 18%
(B) 49% तथा 42%
(B) 49% and 42%
(C) 48% and 44% (C) 48% तथा 44%

(D) 10% and 18% (D) 10% तथा 18%

18. Which one is not an example of 18. कौनसा एक समस्थैतिक (होमियोस्टेसिस) का उदाहरण
homeostasis ? नही हैं? 
(A) Blood osmolarity in mammals (A) स्तनधारीयो में रक्त परासरण
(B) Maintenance of body temperature in (B) मानव में शरीर को ताप बनाये रखना 
human
(C) भालू की शीत निन्द्रा
(C) Winter sleep of bear
(D) Osmoregulation in fishes (D) मछलियों में ऑस्मोरे ग्यूलेशन 

19. Label A , B and identify organism (c) 19. चिन्ह A, B व जीव C को पहचाने

(A) A = Heterocyst B = Mucilagenous (A) A = हेटेरोसिस्ट B = श्लेष्मी आच्छद C = नोस्टोक


sheath C = Nostoc, an archaebacteria एक आर्कि जीवाणु
(B) A = Heterocyst B = Mucilagenous (B) A = हेटेरोसिस्ट B = श्लेष्मी आच्छद C
sheath C = Nostoc
= नोस्टोक 
(C) A = Mucilagenous, B = Heterocyst, C =
Nostoc (C) A = श्लेष्मी आच्छद  B = हेटेरोसिस्ट  C
= नोस्टोक 
(D) A = heterocyst, B = Mucilagenous
sheath, C = Nostoc, a filamentous (D) A = हेटेरोसिस्ट  B =  श्लेष्मी आच्छद  C
algae = नोस्टोक एक तंतुमयी शैवाल
20. In which of the following steps of citric acid 20. साइट्रि क अम्ल चक्र के निम्नलिखित में से किस चरण में,
cycle, CO2 is evolved?          CO2 निर्मित होती है?         
I. Citric acid → α -ketoglutaric acid I. साइट्रि क अम्ल → α-कीटोग्लुटरिक अम्ल
II. Succinic acid Malic acid
II. सक्सिनिक अम्ल → मेलिक अम्ल

III. Malic acid → Oxaloacetic acid


III. मेलिक अम्ल → ऑक्सालोएसिटिक अम्ल
IV. α -ketoglutaric acid → Succinyl Co-A
IV. α-कीटोग्लुटरिक अम्ल → सक्सिनिल Co-A
(A) I and II
(A) I तथा II
(B) I and IV
(B) I तथा IV
(C) II and III
(C) II तथा III
(D) II and IV
(D) II तथा IV

21. Semiconservative DNA replication was 21. अर्द्धसंरक्षी DNA  प्रतिकृ तियन मेसल्सन व स्टाल द्वारा
proved by Messelson and Stahl, in which सिद्ध किया गया जिसमें DNA बनता है
DNA was made
(A) N15 के उपयोग से रे डियोधर्म
(A) Radioactive using N15
(B) N14 भारी
(B) Heavy using N14
(C) 15NH Cl भारी
15NH Cl 4
(C) Heavy using 4
(D) 14NH Cl रे डियोधर्मी
(D) Radioactive using 14NH Cl 4
4

22. Which of the following statement(s) is/are 22. निम्नलिखित में से कौनसे कथन सही नही है?
not correct? (i) कॉर्क कैं बियम को कागजन (phellogen) भी कहा
(i) Cork cambium is also called phellogen. जाता है।
(ii) Cork is also called phellem. (ii) कॉर्क को काग (phellem) भी कहा जाता है। 
(iii) Secondary cortex is also called
(iii) द्वितीयक वल्कु ट को परिचर्म (periderm) कहा
periderm.
(iv) Cork cambium, cork and secondary जाता है।
cortex are collectively called phelloderm. (iv) कॉर्क कैं बियम, कॉर्क तथा द्वितीयक वल्कु ट
मिलकर कागअस्तर (phelloderm) कहलाते है।
(A) (iii) and (iv)
(A) (iii) and (iv)
(B) (i) and (ii)
(B) (i) and (ii)
(C) (ii) and (iii)
(C) (ii) and (iii)
(D) (ii) and (iv)
(D) (ii) and (iv)
23. Succession is a- 23. अनुक्रमण हैं; एक-
(A) Long term process (A) दीर्घ कालिक प्रक्रिया 
(B) Very fast process (B) बहुत तीव्र प्रक्रिया 
(C) Process leading to the development of (C) समष्टि के विकास के लिए अग्रणी प्रक्रिया
a population
(D) प्रवास
(D) Migration
24. In the diagrams given below, different 24. नीचे दिये गये चित्र में, (a), (b), (c), (d) व (e)  के
algae have been labelled as (a), (b), (c), रूप में विभिन्न शैवाल चिन्हित किये गये है। इन शैवाल
(d) and (e). These algae are, respectively, की क्रमशः पहचान करे -
identified as-

(A)
(A) डिक्टिओटा पॉलीसाइफोनिया पोरफाइटा फ्यूकस लेमिनेरिया
Dictyota Polysiphonia Porphyra Fucus Laminaria
(B)
(B) पोरफाइटा डिक्टिओटा लेमिनेरिया फ्यूकस पॉलीसाइफोनिया
Porphyra Dictyota laminaria Fucus PolySiphonia
(C)
(C) डिक्टिओटा पॉलीसाइफोनिया पोरफाइटा लेमिनेरिया फ्यूकस
Dictyota Polysiphonia Porphyra Laminaria Fucus
(D)
(D)
लेमिनेरिया पॉलीसाइफोनिया पोरफाइटा डिक्टिओटा फ्यूकस
Laminaria PolySiphonia porphyra Dictyota Fucus

25. Assertion : When the ambient 25. अभिकथन- जब परिवेश का तापमान अधिक होता है
temperature is high and the soil contains और मिट्टी में जल की अधिकता होती है, तो पौधे
excess of water, plants tend to lose water वातरं ध्रों से बूंदों के रूप में जल का निष्कासन करते हैं।
in the form of droplets from lenticels. कारण- मूल दाब वातरं ध्रों से जल हानि की दर को
Reason : Root pressure regulates the rate नियंत्रित करता है।
of loss of water from lenticels. 
(A) कथन और कारण दोनों सत्य हैं और कारण कथन
(A) Both the assertion and reason are true की व्याख्या करता है
and reason explains the assertion
(B) कथन और कारण दोनों सत्य हैं लेकिन कारण कथन
(B) Both the assertion and reason are true की व्याख्या नहीं करता है
but reason does not explain the assertion
(C) कथन सत्य है परन्तु कारण असत्य है
(C) Assertion is true but reason is false
(D) Both are wrong (D) दोनों गलत हैं

26. What chemical groups are at the end of 26. कौनसा  रासायनिक  समूह  पॉलीन्यूक्लियोटाइड के अंत
poly-nucleotide? मे होता है?
(A) 3'OH (hydroxyl) at one end and 5'-P at (A) 3'OH- (हाइड्रॉक्सील) एक सिरे पर और 5'-P
other end दू सरे पर
(B) Sugar at one end and PO4 at other end
(B) शर्क रा एक सिरे पर और P O4 दू सरे सिरे पर
(C) `A' at one end and G at other end
(C) 'A' एक सिरे पर और 'G'  दू सरे सिरे पर
(D) There is a great variation in the
arrangement (D) यहाँ क्रम में बहुत अधिक विभिन्नताएँ है

27. The amount of biomass or organic matter 27. पौधों द्वारा प्रकाश संश्लेषण के दौरान जैव मात्रा या
produced per unit area over a time period कार्बनिक पदार्थों की मात्रा जा प्रति इकाई क्षत्रफल में एक
by plants during photosynthesis is called- समय के दौरान उत्पादित की गई है, कहलाती है-
(A) Net primary production (A) नेट प्राथमिक उत्पादकता
(B) Secondary production
(B) द्वितीयक उत्पादकता
(C) Primary production
(C) प्राथमिक उत्पादकता
(D) Gross primary production
(D) सकल प्राथमिक उत्पादकता
28. Difference between osmotic pressure and 28. परासण दाब तथा स्फीति दाब में अन्तर है-
turgor pressure is-
(A) DPD
(A) DPD
(B) वाष्पोत्सर्जन खिंचाव 
(B) Transpiration pull
(C) परासण विभव
(C) Osmotic potential 
(D) विलयन विभव
(D) Solution potential
29. In Cycas, specialised roots associated with 29. सायकस में  N2 स्थिर करने  वाले सायनोबैक्टीरिया से
N2 fixing cyanobacteria are referred to as. संयोजित विशिष्ट जड़ो को कहा जाता है -
(A) Tap root (A) मूसला मूल
(B) Coralloid root (B) प्रवाल मूल
(C) Adventitious root
(C) अपस्थानिक मूल
(D) All the above
(D) उपरोक्त सभी

30. The place where RNA-polymerase attaches 30. वह स्थान जहाँ RNA पॉलीामरे ज DNA से सान्द्रता है,
with the DNA is called- कहलाता है
(A) Promoter site (A) उन्नायक स्थल
(B) Operator site (B) चालक स्थल
(C) Activator site
(C) सक्रियक स्थल
(D) Repressor site
(D) संदमन स्थल
31. Accelerated eutrophication involves human 31. मानव भूमिका में सम्मिलित त्वरित यूट्रोफिके शन किसके
role through addition of- संयोजन के माध्यम से होता है-
(A) Sewage into water body (A) जल निकाय मे बाहित मल
(B) Fertilizers into water body (B) जल निकाय में उर्वरक
(C) DDT into water body
(C) जल निकाय में DDT
(D) Both A and B
(D) A व B दोनो

32. Match the following columns.                 32. निम्न कॉलम को सूमेलित करे  
Column-I Column-II कॉलम -I (अनिवार्य कॉलम -II (कमी
   
  (Essential   (Deficiency तत्व) का कारण)
elements) causes) N, K, Mg, S, Fe, कोशिका विभाजन का
(a) (i)
N, K, Mg, S, Fe, Inhibit cell Mn, Zn तथा Mo संदमन
(a) (i)
Mn, Zn and Mo division 
(b) N, K, S तथा Mo (ii) नेक्रोसिस 
(b) N, K, S and Mo (ii) Necrosis
(c) Ca, Mg, Cu तथा K (iii) पुष्पन में विलम्ब
Delay in
(c) Ca, Mg, Cu and K (iii) (d) N, S तथा Mo (iv) क्लोरोसिस
flowering

(d) N, S and Mo (iv) Chlorosis


(A) a-iv, b-ii, c-ii, d-i

(A) a-iv, b-ii, c-ii, d-i (B) a-i, b-ii, c-iii, d-i

(B) a-i, b-ii, c-iii, d-i (C) a-iv, b-i, c-ii, d-iii

(C) a-iv, b-i, c-ii, d-iii (D) a-ii, b-iii, c-iv, d-i

(D) a-ii, b-iii, c-iv, d-i


33. Resistance to cereal leaf beetle in wheat is 33. गेहू में धान्यपर्ण भृंग के प्रति प्रतिरोधकता का कारण है
due to
(A) चिकनी पत्तियों वाला विशेषक
(A) Smooth leaved trait
(B) उच्च एस्पार्टिक अम्ल
(B) High aspartic acid
(C) ठोस तना
(C) Solid stem
(D) रोमिल पत्तिया
(D) Hairy leaves
34. Identify colloidal impurities of domestic 34. घरे लू वाहित मल जल की कोलोइड अशुद्धियों को
sewage water- पहचाने-
(A) Silt (A) सिल्ट 
(B) Phosphate (B) फोस्फोरस पदार्थ 
(C) Calcium
(C) के ल्सियम 
(D) Fecal matter
(D) मल पदार्थ

35. Match the column- 35. निम्न कॉलम को सूमेलित करे -


(a) Auxin (i) Herring sperm DNA (a) ऑक्सिन (i) हेरिं ग स्पर्म DNA
(b) Cytokinin (ii) Inhibitor of growth (b) (ii)
साइटोकाइनिन वृद्धि को रोकना 

(c) Gibberellin (iii) Apical dominance (c) (iii)


जिबरे लिन शीर्ष प्रभाविता
(d) Ethylene (iv) Fruit ripening (d) (iv)
एथिलिन फल का परिपक्वन
Induces amylase (e) (v)
(e) Abscisic acid (v) एब्सिसिक अम्ल एमाइलेज संश्लेषण को प्रेरित करना
synthesis
(A) a-iii, b-i, c-v, d-iv, e-ii
(A) a-iii, b-i, c-v, d-iv, e-ii
(B) a-iv, b-v, c-i, d-iii, e-ii
(B) a-iv, b-v, c-i, d-iii, e-ii
(C) a-ii, b-i, c-v, d-iii, e-iv
(C) a-ii, b-i, c-v, d-iii, e-iv
(D) a-iii, b-i, c-v, d-ii, e-iv
(D) a-iii, b-i, c-v, d-ii, e-iv
Biology

36. An anther is? 36. परागकोश है?


(A) Microsporophyll's fertile part (A) लघु बीजाणुपर्ण का निषेचित भाग
(B) Sporophytic structure (B) बीजाणुधानिक संरचना
(C) Site of development of male (C) नर युग्मकोद्भिद के परिवर्धन का स्थल
gametophyte
(D) उपरोक्त सभी
(D) All of the above
37. The leaf base expands into a sheath 37. पर्णाधार चादर की तरह फै लकर तने को पूरा अथवा
covering the stem partially or wholly. आंशिक रूप से ढक लेता है। 
This is the characteristic of यह किसका लक्षण है-
(A) Dicot
(A) द्विबीजपत्री का
(B) Monocot
(B) एकबीजपत्री का
(C) Pteridophytes
(C) टेरिडोफाइट का
(D) Gymnosperm
(D) जिम्नोस्पर्म का

38. Interfascicular cambium develops from the 38. अन्तरपूलीय एधा किसकी कोशिकाओं से विकसित होती
cells of है:
(A) Medullary rays (A) मज्जा किरणें
(B) Xylem parenchyma
(B) जाइलम मृदुतक
(C) Endodermis
(C) अन्तस्त्वचा
(D) Pericycle
(D) परिरम्भ

39. In operon model for gene regulation, the 39. जीन नियमन के लिए ऑपेरॉन मॉडल में, नियामक जीन
regulator gene regulates chemical reaction कोशिका में रासायनिक अभिक्रिया को नियंत्रित करता है
in the cell by -
(A) inhibiting the transcription of mRNA (A) mRNA के अनुलेखन को रोककर
(B) inactivating enzymes involved in the (B) अभिक्रिया में शामिल एं जाइमो को निष्क्रिय करके
reaction
(C) के न्द्रक विभाजन को रोककर
(C) inhibiting nuclear division
(D) inactivating nuclear division (D) के न्द्रक विभाजन को निष्क्रिय करके

40.  Which of the following pair of enzymes are 40. निम्न् में से एं जाइमो का कौनसा युग्म बोतलबंद फल रस
used in bottled fruit juice industry : उद्योग में उपयोग किया जाता है
(A) Pectinase and Lipase (A) पेक्टिनेज व लाइपेज
(B) Protease and Pectinase (B) प्रोटिऐज व पेक्टिनेज
(C) Amylase and Streptokinase
(C) एमाइलेज व स्ट्रेप्टोकाइनेज
(D) Streptokinase and Lipase
(D) स्ट्रेप्टोकाइनेज व लाइपेज
41. The binding site of t RNA with m RNA and 41. mRNAऔर अमीनो अम्लो के साथ tRNA का बन्धन
amino acids respectively are स्थल क्रमशः है - 
(A) m RNA with DHU loop end and amino (A) mRNAके साथ DHU लूप सिरा और अमीनो अम्ल
acid with CCA end के साथ CCA सिरा 
(B) m RNA with CCA end and amino acid
(B) mRNA के साथ CCA सिरा और अमीनो अम्ल के
with anticodon loop
साथ एन्टीकोडोन लूप 
(C) m RNA with anticodn loop and amino
acid with DHU loop (C) mRNA के साथ एन्टीकोडोन लूप और अमीनो अम्ल
के साथ  DHU लूप 
(D) m RNA with anticodon loop and amino
acid with CCA end (D) mRNA के साथ एन्टीकोडोन लूप और अमीनो अम्ल
के साथ CCA सिरा 

42. A Traditional strategy for the protection of 42. भारत में जैविक विविधता को बचाने के लिए
biodiversity in India : पारम्परिकपद्धति है
(A) national park (A) राष्ट्रीय उद्यान
(B) biosphere reserve (B)  जैवमण्डल रिर्जव
(C) sacred forest & lakes
(C) पवित्र वन व झीलें
(D) all of the above
(D) उपरोक्त सभी
43. Which one is product of aerobic 43. कौनसा एक वायुवीय श्वसन का उत्पाद हैं?
respiration?
(A) मेलिक अम्ल 
(A) Malic acid
(B) एथिल एल्कोहल
(B) Ethyl alcohol
(C) लेक्टिक अम्ल
(C) Lactic acid
(D) पाइरूविक अम्ल
(D) Pyruvic acid
44. In Mosses creeping, green, branched and 44. माॅस में विसर्पी, हरा, शाखित तथा प्रायः तन्तुमय अवस्था
frequently filamentous stage is called :- होती हैः -
(A) Protonema (A) प्रोटोनिमा
(B) Rhizome (B) राइजोम
(C) Rhizophore
(C) राइजोफोर
(D) All of these
(D) उपरोक्त सभी
45. lf a hemophilic man marries a carrier 45. यदि एक हीमोफिलिक पुरूष, वाहक स्त्री से विवाह
woman then which of the following holds करता है तो उनकी संततियों के लिए निम्न में से क्या सही
true for their progenies? होगा?
(A) 50% daughters are carrier and 50% (A) 50% पुत्रियाँ वाहक होंगी और 50% हीमोफिलिक
are hemophilic
होगी।
(B) All the daughters are hemophilic.
(B) सभी पुत्रियाँ हीमोफिलिक होंगी।
(C) All sons are hemophilic and all
daughters are normal. (C) सभी पुत्र हीमोफिलिक और सभी पुत्रियाँ सामान्य
होंगी।
(D) All sons normal, all daughters carriers.
(D) सभी पुत्र सामान्य और सभी पुत्रियाँ वाहक होंगी।
46. What is common to the techniques- 46. इन तकनीकों में क्या समान हैं-
(i) in vitro fertilization (i) इन विट्रो निषेचन
(ii) Cryo presevation and (ii) क्रायोप्रिजर्वेशन औार
(iii) tissue culture
(iii) ऊतक संवर्धन
(A) All are in situ conservation methods.
(A) सभी स्व स्थाने संरक्षण विधियाँ हैं।
(B) All are ex situ conservation methods.
(B) सभी बाह्नय स्थाने संरक्षण विधियाँ हैं।
(C) All require ultra moden equipment and
large space. (C) सभी को अल्ट्रा माॅर्डन उपकरणो और अधिक स्थान
की आवश्यकता होती है।
(D) All are methods of conservation of
extinct organism. (D) सभी विलुप्त जीवों को संरक्षण की विधियाँ हैं।

47. Assertion : Six Molecules of CO2, 12 47. अभिकथन: एक हेक्सोज अणु के निर्माण के लिए  CO2
molecules of NADPH + H+ and eighteen के 6 अणु, NADPH + H+ के 12 अणु और 18
ATP are required to form one hexose ATP की आवश्यकता होती है।
molecule
कारण: प्रकाश अभिक्रिया  ATP और NADPH +
Reason : Light reaction produces ATP and
H+  उत्पन्न करती है।
NADPH + H+
(A) Both the assertion and reason are true (A) अभिकथन और कारण दोनों सत्य हैं और कारण
and reason explains the assertion कथन की व्याख्या करता है

(B) Both the assertion and reason are true (B) अभिकथन और कारण दोनों सत्य हैं लेकिन कारण
but reason does not explain the assertion अभिकथन की व्याख्या नहीं करता है

(C) Assertion is true but reason is false (C) अभिकथन सत्य है परन्तु कारण असत्य है
(D) Assertion is false but reason is true (D) अभिकथन असत्य है परन्तु कारण सत्य है

48. A hereditary disease which is never passed 48. एक वंशानुगत रोग जो पिता से पुत्र में कभी नहीं जाता है-
from father to son is-
(A) ओटोसोमल सहलग्न विकार
(A) Autosomal linked disease
(B) X-गुणसूत्री सहलग्न विकार
(B) X-chromosomal linked disease
(C) Y-गुणसूत्री सहलग्न विकार
(C) Y-chromosomal linked disease
(D) कोई नही
(D) None of the above
49. Most serious threat to wild life comes 49. वन्य जीवो के लिए सबसे गंभीर खतरा आता है-
from-
(A) विदेशी जातियों के परिचय से
(A) Introduction of exotic species
(B) अत्यधिक दोहन से
(B) Over exploitation
(C) अंतर्राष्ट्रीय व्यापार से
(C) International trade
(D) आवास के नष्ट होने से
(D) Habitat destruction
50. N2 + 8e– + 8H+ + 16ATP → 2NH3 + H2 + 50. N2 + 8e– + 8H+ + 16ATP → 2NH3 + H2 +
16 ADP + 6Pi 16 ADP + 6Pi
The above equation refers to- उपरोक्त समीकरण दर्शाती हैं-
(A) Ammonification (A) अमोनीकरण
(B) Nitrification (B) नाइट्रीकरण
(C) Nitrogen fixation (C) नाइट्रोजन स्थिरीकरण
(D) Denitrification
(D) विनाइट्रीकरण
Biology

51. Which of the following match is correct? 51. निम्न में से कौन - सा मिलान सही है?
(a) Common cold - Droplet infection (a) जुकाम (common cold) - ड्रोपलेट संक्रमण
(b) Typhoid - Contaminated food
and (Droplet infection)
water

(b) टायफाइड (Typhoid) - दू षित


(c) AIDS - Shaking hands



(contaminated) भोजन व जल 
(d) Ringworm - Using infected towels
(c) ऐड्स (AIDS) - हाथ मिलाना
(Shaking
(A) a and b hands)
(d) दाद
(Ringworm) - संक्रमित तौलिये
(B) c and d
(infected towels) का प्रयोग
(C) a and c
(A) a तथा b
(D) a, b and d
(B) c तथा d

(C) a तथा c

(D) a, b तथा d

52. Assertion : When pH of blood increase 52. अभिकथन: रूधिर की pH बढ़ने पर O2 वियोजन वक्र
then oxygen dissociation curve shifts दाँयी ओर विस्थापित हो जाता है।
towards right side.  कारण: उच्च pH के कारण O2
की Hb के साथ बन्धुता
Reason : Due to high pH affinity of O2 कम हो जाती है।
with Hb decreases. 
(A) दोनों (A) एवं (R) सही है लेकिन (R), (A) की
(A) Both (A) and (R) are true and (R) is
the correct explanation of (A) सही व्याख्या है।

(B) Both (A) and (R) are true but (R) is (B) दोनों (A) एवं (R) सही है लेकिन (R), (A) की


not the correct explanation of (A) सही व्याख्या नहीं है।

(C) (A) is true statement but (R) is false. (C) (A) सही है लेकिन (R) गलत है।
(D) Both (A) and (R) are false. (D) (A) तथा (R) दोनों ही असत्य है।

53. Choose the incorrect match from the 53. निम्नलिखित में से गलत मिलान का चयन कीजिए-
following-
(A) ह्यूगो डी व्रीस - उत्परिवर्तन सिद्धांत
(A) Hugo de Vries - Mutation theory
(B) चार्ल्स डार्विन - प्राकृ तिक चयन सिद्धांत
(B) Charles Darwin - Natural selection
theory (C) अल्फ्रे ड वालेस - पैन्सपर्मिया का सिद्धांत
(C) Alfred Wallace - Theory of (D) लैमार्क -  ऊपार्जित लक्षणों की वंशानुगति
panspermia
(D) Lamarck - Inheritance of acquired
characters
54. Which one of the following is wrongly 54. निम्न में से कौनसे गलत सुमेलित है ? 
matched ?
(A) प्रोजेस्टेरॉन - कॉर्पस ल्यूटियम को व्यवस्थित करता
(A) Progesterone — Maintains corpus है
Luteum
(B) ऐल्डोस्टोरॉन - Na+ का पुनः अवशोषण तथा
(B) Aldosterone — Na+ reabsorption and
K+ का उत्सर्जन
K+ excretion
(C) Oxytocin —Uterine contraction & milk (C) ऑक्सीटोसिन - गर्भाशय संकु चन तथा दुग्ध स्त्रवण
ejection (D) ग्लूकोकार्टिकोइड - यह प्रतिशोथ हार्मोन हैं।
(D) Glucocorticoids— Anti-inflammatory
hormone
55. Which one of the following statement is 55. निम्नलिखित में से कौन-सा कथन असत्य है ?
incorrect?
(A) एसीडिया में, पृष्ठ रज्जु सिर से पूंछ तक फै ली रहती
(A) In Ascidia, notochord extends from हैं जो जीवन के अंत तक बनी रहती हैं।
head to tail region and persist throughout
the life. (B) टीनोफोर  में चलन/ गमन पक्ष्माभी कॉम्ब प्लेट द्वारा
होता है।
(B) In ctenophors, locomotion is mediated
by cilliary comb plates. (C) फै शिओला में फ्लेम कोशिकाएँ (flame cells)
(C) In Fasciola, flame cells take part in उत्सर्जन में सहायक होती हैं।
excretion.
(D) ओबेलिया में एक्टोडर्म तथा एं डोडर्म के मध्य
(D) Mesoglea is present in between मिसोग्लिया उपस्थित होती हैं।
ectoderm and endoderm in Obelia.
56. How many of the following diseases are 56. निम्नलिखित में से कितने रोग वाहक जनित रोग है ?
vector borne diseases ?
डेंगु, चिकनगुनिया, फाइलेरिएसिस, ऐस्के रिएसिस,
Dengue, Chikungunya, Filariasis, टायफॅाइड, मलेरिया, न्युमोनिया
Ascariasis, Typhoid, Malaria, Pneumonia 
(A) तीन
(A) Three  
(B) चार
(B) Four
(C) Five (C) पाँच

(D) Six (D) छः

57. Which of the following are actively 57. निम्नलिखित में से निस्पंद से रिनल नलिकाओं द्वारा क्या
reabsorbed by renal tubules from filtrate - सक्रिय रूप से पुन: अवशोषित होते हैं -
(A) Na
+
,K
+
,H
+
(A) Na
+
,K
+
,H
+

(B) H
+
,  K
+
,  Ammonia (B) H
+
,  K
+
,  अमोनिया
(C) Glucose, Na ,  Amino acid +

(C) ग्लूकोज, Na , एमीनो अम्ल +

(D) H2 O , Urea, Glucose


(D) H2 O , यूरिया, ग्लूकोज

58. Animal husbandry deals with 58. पशुपालन का संबंध किससे है:-
I. Breeding of livestock buffaloes, cows, I. पशुधन का संबंध भैंस, गाय, भेड़, ऊँ ट आदि के
sheep, camels, etc., that are useful to प्रजनन से है जो मानव के लिए लाभप्रद हैं। 
humans. II. मछलियो, मोलस्क तथा क्रस्टेशियन का पालन पोषण,
उनको पकड़ना बेचना आदि शामिल है। 
II. Rearing, catching, selling, etc., of fish,
III. मानव उपयोग के लिए कु क्कु ट प्रजनन
molluscs and crustaceans.
दिये गये उपरोक्त कथनों में से कौनसा/से सही है ? 
III. Breeding of fowls for human use.
(A) I तथा II
Which of the statement give above are
correct ? (B) I तथा III

(A) I and II (C) II तथा III

(B) I and III (D) I, II तथा III


(C) II and III
(D) I, II and III
59. Identify correct statement : 59. सही कथनो की पहचान करें  :
(i) Ileum is the highly coiled part of the (i) इलियम छोटी आंत का अत्यधिक कुं डलित भाग है।
small intestine.
(ii) म्यूकोसा स्तर आमाशय में अनियमित वलय (रुघे)
(ii) The mucosa forms irregular folds एवं छोटी आंत में अंगुलीनुमा प्रवर्ध बनाता है जिसे अंकु र
(rugae) in the stomach and small finger -
(विलाई) कहते हैं
like foldings called villi in the small
intestine. (iii) ग्रसिका का अमाशय में खुलना एक पेशीय
(iii) A muscular sphincter regulates (अमाशय - ग्रसिका) अवरोधिनी द्वारा नियंत्रित होता है।
opening of oesophagus into the stomach.
(iv) सिरोसा सबसे बाहरी परत है और एक पतली
(iv) Serosa is the outermost layer made up मेजोथिलियम और कु छ संयोजी ऊतकों से बनी होती है।
of thin mesothelium and some connective
tissue. (A) (i), (ii) तथा (iv)

(A) (i),  (ii)  and  (iv)


(B) (ii), (iii) तथा (iv)

(B) (ii),  (iii)  and  (iv) (C) (i), (iii) तथा (iv)

(C) (i),  (iii)  and  (iv) (D) सभी सही है।

(D) All   are   correct

60. Identify the incorrect set of statements : 60. निम्न में से गलत कथनों के समूह की पहचान करें :
(a) Spermatids are transformed into (a) शुक्राणुप्रसू का शुक्राणुजन में परिवर्तन की प्रकिया
spermatogonia by the process called शुक्राणुजनन (स्पर्मियोजेनेसिस) कहलाती है।
spermiogenesis.
(b) तृतीयक पुटक का एक विशिष्ट लक्षण तरल से भरी
(b) Tertiary follicle is characterised by a
गुहा है जिसे गहवर (एं ट्र म) कहा जाता है ।
fluid filled cavity called antrum.
(c) Oogenesis is initiated during embryonic (c) अंडजनन की शुरुआत भ्रुणीय परिवर्धन के चरण के
development stage. दौरान होती हैं।

(d) Each primary oocyte gets surrounded (d) प्रत्येक प्राथमिक ऊसाइट ज़ोना पेलुसीडा की एक
by a layer of zona pellucida and is called परत से घिरा होता है और इसे प्राथमिक पुटक कहा जाता
the primary follicle. है।
(A) (a) and (b) (A) (a) तथा (b)
(B) (b) and (c) (B) (b) तथा (c)
(C) (a) and (d)  (C) (a) and (d)
(D) (c) and (d) (D) (c) तथा (d)
61. Find out the correct statements. 61. निम्न कथनों को पढ़िए और सही कथनों का चुनाव करें ।
(a) Squamous epithelium are involved in (a) शल्की उपकला एक विसरण सीमा बनाने जैसे कार्यों
functions like forming a diffusion boundary. में शामिल है।
(b) Cuboidal epithelium found in the lining (b) घनाकार उपकला आंत तथा आमाशय की आस्तर
of stomach and intestine.
पर पायी जाती है।
(c) The cillated columnar epithilium is
found in ducts of glands and tubular parts (c) पक्ष्मांभी स्तंभाकार उपकला गुर्दों में नेफ्रॉन के
of nephron. नलिकाकार भागों तथा ग्रंथियों की वाहिनियों में पायी जाती
है।
(d) In epithelial tissue, cells are compactly
packed with little intercellular matrix. (d) उपकला ऊतक में, कोशिकाए अंतरकोशिकीय
आधात्री द्वारा द्रढतापूर्वक जुड़ी रहती हैं।
(e) The simple cuboidal epithelium
provides protection against chemical and (e) सरल घनाकार उपकला रासायनिक और यांत्रिक
mechanical stresses. प्रतिबलो के खिलाफ सुरक्षा प्रदान करती है।
(A) (b), (c) and (e) only (A) के वल (b), (c) और (e)
(B) (a) and (d) only
(B) के वल (a) और (d) 
(C) (a), (b) and (e) only
(C) के वल (a), (b) और (e)
(D) (a), (b), (d) and (e) only
(D) के वल (a), (b), (d) और (e)
62. Which of the following statements are 62. निम्न कथनों में से कौन - सा कथन पेशी प्रोटीन्स के संदर्भ
correct regarding muscle proteins? में सही है?
(i) Actin is a thin filament and is made up (i) एक्टिन एक पतला तंतु है और दो F- एक्टिन्स का
of two strands of F - actins बना होता है
(ii) The complex protein, tropomyosin is
(ii) जटिल प्रोटीन, ट्रोपोमायोसिन ट्रोपोनिन पर नियमित
distributed at regular intervals on the
troponin अन्तरालों पर वितरित होता है
(iii) Myosin is a thick filament which is also (iii) मायोसिन एक मोटा तंतु है, जो एक बहुलक प्रोटीन
a polymerized protein भी है
(iv) The globular head of meromyosin (iv) मेरोमायोसिन का गोलाकार सिर हल्के मेरोमायोसिन
consists of light meromyosin (LMM). (LMM) का बना होता है
(A) (i), (ii) and (iii) (A) (i), (ii) तथा (iii)
(B) (i), (ii) and (iv) (B) (i), (ii) तथा (iv)
(C) (i) and (iii) (C) (i) तथा (iii)
(D) (ii) and (iv)
(D) (ii) तथा (iv)

63. Match the columns and choose the correct 63. कॉलम का मिलान करें और सही विकल्प चुनें-
option-
   कॉलम-I   कॉलम -II
  Column-I   Column-II  (a) होमो हैबिलिस (i) पूर्वी अफ्रीकी घास के मैदान
(a) Homo habilis (i) East African grasslands (b) होमो इरे क्टस (ii) पहला मानव जैसा होमिनिड
First human-like being (c) ऑस्ट्रेलोपेथिकस (iii) 1,00,000-40,000 साल पहले
(b) Homo erectus (ii)
hominid
(d) होमो निएं डरथल (iv) कपाल क्षमता लगभग 900CC
(c) Australopithecus (iii) 1,00,000-40,000 years back
Homo Cranial capacity around
(d)
neanderthalensis
(iv)
900CC
(A) a–(i), b–(ii), c–(iii), d–(iv)

(A) a–(i), b–(ii), c–(iii), d–(iv) (B) a–(ii), b–(i), c–(iv), d–(iii)

(B) a–(ii), b–(i), c–(iv), d–(iii) (C) a–(ii), b–(iv), c–(i), d–(iii)

(C) a–(ii), b–(iv), c–(i), d–(iii) (D) a–(ii), b–(iv), c–(iii), d–(i)

(D) a–(ii), b–(iv), c–(iii), d–(i)


64. Which of the following is an amino acid 64. निम्नलिखित में से कौन-सा हार्मोन ऐमीनो अम्ल से व्युत्पन्न
derived hormone ? होता है ?
(A) Aldosterone (A) एल्डोस्टीरोन
(B) Epinephrine (B) एपिनेफ्रीन
(C) Estradiol
(C) एस्ट्राडिऑल
(D) ADH
(D) ADH
65. Statement -I : Contraceptive pills inhibits 65. कथन-I :  गर्भनिरोधक गोलियां अंडोत्सर्जन और
ovulation and implantation as well as alters अंतर्रोंपण को संदमित करने के साथ-साथ गर्भाशय ग्रीवा
the quality of cervical mucus. की श्लेष्मा को गुणता को भी बदल देती हैं।
Statement -II : LNG-20 and Progestasert कथन-II :  LNG  -20 तथा प्रोजेस्टासर्ट गर्भाशय  को
make the uterus unsuitable for
भ्रूण के रोपण के लिए अनुपयुक्त बनाते हैं तथा गर्भाशय
implantation and the cervix hostile to the
sperms. ग्रीवा को शुक्राणुओं का विरोधी बनाते हैं।

(A) Both statements I and II are correct. (A) कथन I व II दोनो सही है। 

(B) Both statements I and II are incorrect. (B) कथन I व II दोनो गलत है। 

(C) Only statement I is correct. (C) के वल कथन I सही है। 


(D) Only statement II is correct. (D) के वल कथन II सही है। 
66. Given below are two statements :  66. नीचे दो कथन दिये गये है-
Statement-I : The reflex pathway कथन-I : प्रतिवर्ती क्रिया पथ कम से कम एक अभिवाही
comprises at least one afferent neuron and न्यूरॉन और एक भिन्न न्यूरॉन, जो कि निश्चित क्रम में नही
one different neuron not appropriately होते है, से बना होता हैं। 
arranged in a series. 
कथन-II :अभिवाही न्यूरॉन संवेदी अंगो से संके त ग्रहण
Statement-II : The afferent neuron
receives signal from a sensory organ and करके पृष्ठ तंत्रिकीय मूल के द्वारा कें दीय तंत्रिका तंत्र में
transmits the impulse via a dorsal nerve आवेगो का संप्रेषण करता है। 
root into the CNS. 
उपरोक्त दिये गये कथनो के आलोक में नीचे दिए गए
In the light of the above statements,
विकल्पों में से सही उत्तर का चयन कीजिए।
choose the most appropriate answer from
the options given below : (A) दोनों कथन I एवं कथन II गलत है।
(A) Both statement I and statement (B) कथन I सही है लेकिन कथन II  गलत है।
II are incorrect
(C) कथन I गलत है लेकिन कथन II सही है।
(B) Statement I is correct but statement
II is incorrect (D) दोनों कथन I एवं कथन II सही है।
(C) Statement I is incorrect
but statement II is correct
(D) Both statement I and statement
II are correct.
67. Person with blood group AB is considered 67. AB रुधिर वर्ग वाले व्यक्ति को सार्वत्रिक ग्राही
as universal recipient because he has : (universal recipient) कहते हैं, क्योंकि:
(A) Both A and B antigens on RBC but no (A) इसकी RBC में A तथा B दोनों प्रतिजन उपस्थित
antibodies in the plasma. हैं, परन्तु इसके प्लाज्मा में कोई प्रतिरक्षी नहीं है।
(B) Both A and B antibodies in the plasma
(B) इसके प्लाज्मा में A तथा B दोनों प्रतिरक्षी
(C) No antigen on RBC and no antibody in (antibody) उपस्थित हैं।
the plasma
(C) न तो इसकी RBC पर और न ही प्लाज्मा पर कोई
(D) Both A and B antigens in the plasma प्रतिरक्षी उपस्थित हैं।
but no antibodies.
(D) इसके प्लाज्मा में A तथा B दोनों प्रतिजन उपस्थित
परन्तु कोई प्रतिरक्षी उपस्थित नहीं।

68. Statement I: In Jaundice, Liver is 68. कथन I : पीलिया (Jaundice) में यकृ त प्रभावित होता
affected,skin and eyes turn yellow due to है, त्वचा और आँखे पित्त वर्णकों के जमा होने से पीले रं ग
the deposit of bile pigments. के दिखाई देते है।
Statement II: Kwashiorkar results from कथन II : क्वाशिओरकर 1 वर्ष से अधिक आयु के
the replacement of mother's milk by a high बच्चों का पोषण माँ के दू ध के स्थान पर उच्च् कै लोरी
calorie - low protein diet in a child more परं तु अल्प प्रोटीन वाला आहार देने से होता है।
than one year in age.
(A) कथन I और कथन II दोनों गलत हैं 
(A) Both statement I and Statement II are
incorrect (B) कथन I सही है लेकिन कथन II गलत है 

(B) Statement I is correct but statement II (C) कथन I गलत है लेकिन कथन II सही है
is incorrect
(D) कथन I और कथन II दोनों सही हैं
(C) Statement I is incorrect but statement
II is correct
(D) Both statement I and Statement II are
correct
69. From the given list, select the number of 69. दी गई सूची से हार्मोनों की संख्या का चयन कीजिए जो
hormones which are not secreted by अपरा द्वारा स्त्रावित नही होते है –
placenta: FSH, LH, hCG, hPL,  इस्ट्रोजन, प्रोजेस्टीरोन,
FSH, LH, hCG, hPL, estrogen, कोर्टिसोल, प्रोलेक्टिन, थायरोक्सिन, रिलेक्सिन
progesterone, cortisol, prolactin, thyroxine,
relaxin.  (A) तीन
(A) Three (B) चार
(B) Four (C) पाँच
(C) Five
(D) छः
(D) Six
70. Given below is the diagrammatic 70. नीचे एक एक्सोन से आवेग संचरण का आरे खित प्रदर्शन
representation of impulse conduction दिया गया है।
through an axon.

         

स्थान A एवं B पर आवेग संचरण से सम्बन्धित गलत


Select the option with the incorrect सुचनाएँ देने वाले विकल्प को चुनिये ?
information about impulse conduction at
(A) सर्कि ट को पूर्ण करने के लिए भितरी सतह पर विधुत
point A and B. :
धारा का प्रवाह स्थान A से B जबकि बाहरी सतह पर
(A) An electric current flows on the inner स्थान B से A की ओर होता है।
surface from site A to B and on outer
surface, from site B to A to complete the (B) स्थान.A पर पैदा हुआ क्रियात्मक विभव स्थान.B पर
circuit पहुँच जाता है।
(B) Action potential generated at site-A (C) यह धारा स्थान B पर आयन चैनलो को खुलने के
arrives at site B लिए प्रेरित करती है ।
(C) This current promotes opening of  (D) B-.स्थान पर ध्रुवणता विपरीत हो जाती है एवं इसे
channels at site-B. पुनः ध्रुवित कहा जाता है।
(D) Polarity at the site-B is reversed and
hence called repolarisation
71. Select the incorrect statement from the 71. निम्न में से असत्य कथन का चयन कीजिए :
following :
(A) कई प्रोटीनों की जैविक गतिविधि के लिए तृतीयक
(A) Tertiary structure is absolutely संरचना अत्यंत आवश्यक है।
essential for biological activity of many
proteins. (B) RNA  में, फॉस्फे ट समूह और पेंटोज शर्क रा के
मध्य का बंधन एक ग्लाइकोसिडिक बंधन है।
(B) In RNA, the bond between the
phosphate group and pentose sugar is a (C) DNA की रीढ़ शर्क रा फॉस्फे ट श्रृंखलाओं से बनी
glycosidic bond. होती है।
(C) Backbone of DNA is made up of sugar
(D) सेल्यूलोज β-1,4 ग्लाइकोसिडिक बंध से बना
phosphate chains.
अशाखित पॉलीसेके राइड हैं।
(D) Cellulose is unbranched polysaccharide
with β-1,4 glycosidic bonds
72. Read the following statements and find out 72. निम्नलिखित कथनों को पड़कर असत्य कथन छांटियेः
the incorrect statement : 
(A) कु त्ता मछली/डॉग फिश (dog fish) में दाँत
(A) Dog fish have teeth that are modified (teeth) उपस्थित होते हैं, जो रूपान्तरित (modified)
placoid scales प्लेकाइड स्के ल्स (placoid scales) होते हैं।
(B) In fighting fish gills are covered by
(B) फाइटिंग फिश (fighting fish) में क्लोम (gills)
operculum while in sting ray gill cover is
absent प्रच्छद (operculum) से ढके रहते हैं परन्तु स्टिंग - रे
(sting ray) में क्लोम आवरण अनुपस्थित होता है।
(C) Air bladder is present in saw fish which
regulates buoyancy while in Angel fish , air (C) आरा मछली/सा- फिश (saw fish) में वायुकोष
bladder is absent उपस्थित होता है जो उत्प्लावकता (buoyancy) का
नियमन करता है। जबकि ऐंजल फिश (Angel fish) में
(D) Mouth of flying fish is terminal while
that of great white shark is ventral वायुकोष अनुपस्थित होता है।

(D) फ्लाइंग फिश (flying fish) में मुख अग्रस्थ


(terminal) परन्तु शार्क (shark) में अधरीय
(ventral) होता है।
73. Read the following statements and choose 73. निम्नलिखित कथनों को पढ़िए और सही विकल्प का चयन
the correct option- कीजिए-
A. The skull of baby chimpanzee is more A. शिशु चिंपैंजी की खोपड़ी अधिक मानव सम है
like an adult human skull than adult अपेक्षाकृ त वयस्क चिंपैंजी की खोपड़ी के ।
chimpanzee skull.
B. प्रत्येक जनसंख्या में अभिलाक्षणिक विभिन्नतायें निहित
B. Any population has built in variations in
their characteristics. होती रही है।

(A) Both statements are correct (A) दोनों कथन सही हैं

(B) Both statements are incorrect (B) दोनों कथन गलत हैं

(C) Only statement A is correct (C) के वल कथन A सही है

(D) Only statement B is correct (D) के वल कथन B सही है


74. Which of the following cannot be detected 74. एम्नियोसेंटेसिस (उल्बवेधन) द्वारा विकसित हो रहे भ्रूण
in developing foetus by amniocentesis? में निम्नलिखित में से किसका पता नहीं लगाया जा सकता
(A) Jaundice है ?

(B) Haemophilia  (A) पीलिया

(C) Sex of foetus (B) हीमोफिलिया

(D) Down syndrome (C) भ्रूण के लिंग

(D) डाउन सिंड्रोम

75. The membrane of the erythrocyte has 75. मनुष्य की रूधिराणु (erythrocyte) की झिल्ली में
approximately _____% of protein  and लगभग_____% प्रोटीन व______% लिपिड मिलता
____% lipids. है। 
(A) 42, 50 (A) 42, 50
(B) 52, 40 (B) 52, 40
(C) 50, 50 (C) 50, 50
(D) 60, 40 (D) 60, 40
76. Identify the components labelled A, B, C 76. दिए गए पक्ष्माभ/कशाभिका के अनुभाग में विभिन्न भागो
and D in the given section of cilia/flagella को दर्शाते हुए A, B, C तथा D नामांकित घटको को
showing different parts. पहचानिए।
Choose the option which shows the उस विकल्प को चुनिए जो भागो का सही नामांकन
correct labelling of parts.
प्रदर्शित करता है। 

(A) A-Plasma membrane, B-Interdoublet


bridge, C-Central microtubule, D-Radial
spoke (A) A-जीवद्रव्य झिल्ली, B-अंतराद्विक सेतु, C-के न्द्रीय
(B) A-Plasma membrane, B-Arm, C-Central सूक्ष्म नलिका, D-अरीय दण्ड
microtubule, D-Radial spoke (B) A-जीवद्रव्य झिल्ली, B-भुजा, C-के न्द्रीय सूक्ष्म
(C) A-Plasma membrane, B-Interdoublet नलिका, D-अरीय दण्ड
bridge, C-Hub, D-Radial spoke
(C) A-जीवद्रव्य झिल्ली, B-अंतराद्विक सेतु, C-धुरी
(D) A-Plasma membrane, B-Interdoublet (Hub), D-अरीय दण्ड ​
bridge, C-Hub, D-Arm
(D) A-जीवद्रव्य झिल्ली, B-अंतराद्विक सेतु, C-धुरी
(Hub), D-भुजा
77. Match List-I with List-II. 77. सूची-I को सूची-II से सुमेलित कीजिए।
  List-I   List-II   सूची-I   सूची-II
(a) Cristae (i) Primary constriction in chromosome (a) क्रिस्टी  (i) गुणसूत्र में प्राथमिक संकीर्णन
(b) Thylakoids (ii) Disc-shaped sacs in golgi apparatus (b) थायलेकॉयड (ii) गॉल्जी उपकरण में डिस्क आकार की थैलिया
(c) Centromere (iii) Infoldings in mitochondria
(c) गुणसूत्र बिन्दु  (iii) सूत्रकणिका में अंतर्वलन
Flattened membranous sacs in stroma
(d) Cisternae (iv) (d) सिस्टर्नी (iv) लवक की पीठिका में चपटी झिल्ली नुमा थैलिया
of plastids
Choose the correct answer from the नीचे दिए गए विकल्पो में से सही उत्तर चुनिए-
options given below-
(A) a-(iv), b-(iii),c-(ii), d-(i)
(A) a-(iv), b-(iii),c-(ii), d-(i)
(B) a-(i), b-(iv),c-(iii), d-(ii)
(B) a-(i), b-(iv),c-(iii), d-(ii)
(C) a-(iii), b-(iv),c-(i), d-(ii)
(C) a-(iii), b-(iv),c-(i), d-(ii)
(D) a-(ii), b-(iii),c-(iv), d-(i)
(D) a-(ii), b-(iii),c-(iv), d-(i)
78. Which of the following statements are 78. निम्न में से कौनसा कथन गलत है
incorrect ?
(a) वायरस व वाइरॉइड को जगत मोनेरा में व्हिटेकर
(a) Viruses & viroids are classified by द्वारा वर्गीकृ त किया गया 
Whittaker in kingdom Monera.
(b) वायरस अ-कोशिकीय जीव होते है
(b) Viruses are non-cellular organisms
(c)  वायरस-विषैला है तथा डब्ल्यू एम. स्टानले ने नाम
(c) Virus-mean venom and name was
coined by W.M. Stanley. दिया 
(d) Contagium vivum fluidum was called (d)  संक्रामक जीवित तरल (कं टोजियम वाइवम
by D.J. Ivanowsky. फ्लुयइडम) डी. जे. इवानोवस्की द्वारा कहा गया 
(e) Viruses are obligate parasite. (e) वायरस अविकल्पी परजीवी है
(f) Virus contains either DNA or RNA. (f) वायरस में या तो DNA या RNA होता है
(A) (a), (b) and (c) (A) (a), (b) तथा (c)
(B) (b), (d) and (f) (B) (b), (d) तथा (f)
(C) (a), (c) and (d) (C) (a), (c) तथा (d)
(D) (d), (e) and (f)
(D) (d), (e) तथा (f)
79. Crossing over takes place between which 79. जीन विनिमय (Crossing over) कौनसे क्रोमेटिड के
chromatids and in which stage of the cell बीच तथा कोशिका चक्र की किस अवस्था में होता है?
cycle?
(A) प्रोफे ज-I की जाइगोटीन अवस्था में असमजात
(A) Non-sister chromatids of non- गुणसूत्रो के  असंतति अर्धगुणूसूत्रो (Non-sister
homologous chromosomes at  Zygotene
chromatids)  के बीच
stage of  prophase I
(B) Non-sister chromatids of homologous (B) प्रोफे ज-I की पेकाइटीन अवस्था में समजात गुणसूत्रो
chromosomes at Pachytene stage के असंतति अर्धगुणसूत्रो (Non-sister chromatids)
of  prophase I के बीच

(C) Non-sister chromatids of homologous (C) प्रोफे ज-I की जाइगोटीन अवस्था में समजात गुणसूत्रो
chromosome at Zygotene stage के असंतति अर्धगुणसूत्रो (Non-sister chromatids)
of  prophase I के बीच
(D) Non-sister chromatids of non- (D) प्रोफे ज-I की पेकाइटीन अवस्था में असमजात
homologous chromosomes at Pachytene गुणसूत्रो के असंतति अर्धगुणसूत्रो (Non-sister
stage of prophase I
chromatids) के बीच
80. Noble prize winner  (2006) for the work on  80. सूत्रकृ मि में  RNAi पर कार्य के लिए नोबल पुरस्कार
RNAi in the nematode is/are- विजेता (2006) है / हैं-
(A) Andrew Fire (A) एं ड्रू फायर
(B) Craig C. Mello (B) क्रे ग सी मेलो
(C) Both (A) and (B)
(C) दोनों (A) और (B)
(D) Kary Mullis
(D) कै री मुलिस
81. Centrioles undergo duplication during (i) of 81. तारकके न्द्र में (ii) की (i)  के दौरान द्विगुणन होता है
(ii) and begin to move towards opposite तथा (iv) की (iii) अवस्था के दौरान कोशिका के
poles of the cell during (iii) stage of (iv). विपरित ध्रुवो की गति प्रारम्भ कर देते हैं।
(A) S-phase, Interphase, Prophase, Mitosis (A) S - प्रावस्था, अन्तरावस्था, पूर्वावस्था, समसूत्रण
(B) S-phase, Interphase, Anaphase,
(B) S - प्रावस्था, अन्तरावस्था, पश्चावस्था, समसूत्रण
Mitosis
(C) Prophase, Mitosis, Metaphase, Mitosis (C) पूर्वावस्था, समसूत्रण, मध्यावस्था, समसूत्रण

(D) Prophase, Mitosis, Anaphase, Mitosis (D) पूर्वावस्था, समसूत्रण, पश्चावस्था, समसूत्रण

82. The basic procedure involved in the 82. r-DNA बनाने के कु छ पद नीचे दिये गये हैं। इस विधिमें
synthesis of recombinant DNA molecule is क्या गलती की गयी है-
depicted below. The mistake in the
procedure is-

(A) पॉलीमरे ज एन्जाइम सम्मिलित नहीं किया गया।

(A) Enzyme polymerase is not included. (B) मैमेलियन डी. एन. ए. को द्विलड़ी बताया गया है।
(B) The mammalian DNA is shown double (C) दो अलग-अलग रे स्ट्रि क्शन एन्जाइम उपयोग में लिये
stranded. गये है।
(C) Two different restriction enzymes are (D) के वल एक ही खण्ड निवेशित किया गया है।
used.
(D) Only one fragment is inserted.
83. A piece of DNA, cut by a restriction 83. DNA के एक खण्ड को रे स्ट्रीक्शन एन्जाइम द्वारा काटने
enzyme, forms bonds with other DNA पर, अन्य DNA अणुओ से बंध का निर्माण  करता है 
molecules which have- जिसमे होते है-
(A) methyl groups attached to them (A) उनको जोडने के मेथिल समूह 
(B) unpaired bases
 
(B) अयुग्मित क्षार 
(C) plasmid components
(C) प्लाज्मिक अवयव 
(D) been fragmented by the same
restriction enzyme (D) एक ही रे स्ट्रीक्शन एन्जाइम द्वारा विखण्डन 

84. Read the following four statements (A-D) 84. नीचे दिये जा रहे चार कथनों (A-D) को इनमेें से दो में
about certain mistakes in two of them- कु छ गलतियाँ ढूं ढ़ने के लिए पढ़िये-
(A) The first transgenic buffalo, Rosie (A) प्रथम पारजीनी भैंस “रोजी” ने ऐसा दू ध पैदा किया
produced milk which was human alpha- जा मानव ऐल्फा-लैक्टैल्बुमिन से भरपूर था।
lactalbumin enriched.
(B) रे स्ट्रि क्शन एं जाइमों का उपयोग DNA को अन्य
(B) Restriction enzymes are used in
isolation of DNA from other macro वृहद् अणुआ से पृथक करने में किया जाता है।
molecules. (C) डाउन-स्ट्रीम (अनुप्रवाह) संसाधन करना R-DNA
(C) Downstream processing is one of the प्रौद्योगिकी का एक चरण है।
steps of R-DNA technology.
(D) निष्क्रियकृ त रोगजनक वेक्टरों का उपयोग R-DNA
(D) Disarmed pathogen vectors are also को परपोषी के भीतर स्थानांतरित करने में भी किया जाता
used in transfer of R-DNA into the host. है।
Which are the two statements having गलतियों वाले दो कथन कौनसे है?
mistakes?
(A) कथन (A) तथा (B)
(A) Statements (A) and (B)
(B) कथन (B) तथा (C)
(B) Statements (B) and (C)
(C) Statements (C) and (D) (C) कथन (C) तथा (D)

(D) Statements (A) and (C) (D) कथन (A) तथा (C)

85. Read the following statements : 85. निम्नलिखित कथनों को पढ़ेः


(i) RNAi involves silencing of specific mRNA (i) RNAi में किसी पूरक dsRNA द्वारा विशिष्ट
due to complementary dsRNA. mRNA का निष्क्रियकरण सम्मिलित होता है
(ii) RNAi takes place in all prokaryotic (ii) RNAi सभी प्रोके रियोटिक जीवो में कोशिकीय सुरक्षा
organism as a method of cellular defense.
की एक विधि के रूप में होता है
(iii) The source of complementary RNA
could be from an infection by viruses (iii) पूरक आरएनए का स्त्रोत आरएनए जीनोम या
having RNA genome or transposons that ट्रान्सपोजोन रखने वाले विषाणु के संक्रमण से हो सकता
replicate via an DNA intermediate. है जो डीएनए मध्यवर्ती के माध्यम से प्रतिकू ट होते है
Select the incorrect statement. गलत कथन का चयन करे -

(A) i, iii (A) i, iii


(B) i, ii (B) i, ii
(C) ii, iii (C) ii, iii
(D) i, ii, iii (D) i, ii, iii
Biology

86. Which of the following statement is 86. निम्नलिखित में से कौनसा कथन गलत है। 
incorrect.
(A) बाह्य अंतरापर्शुक पेशियों का संकु चन पसलियों और
(A) The contraction of external inter-costal उरोस्थि को ऊपर उठा देता है।
muscles lifts up the ribs and the sternum.
(B) हमारे पास अंतः श्वसन और निः श्वसन की शक्ति
(B) We don't have the ability to increase
बढ़ाने की क्षमता नही है।
the strength of inspiration and expiration.
(C) Expiration takes place when the intra (C) निः श्वसन तब होता है, जब आंतर फु फ्फु सी दाब
pulmonary pressure is higher than the वायुमंडलीय दाब से अधिक होता है।
atmospheric pressure.
(D) डायाफ्राम का संकु चन अग्र पश्च अक्ष में वक्ष गुहा का
(D) Contraction of diaphragm increases the आयतन बढ़ा देता है। 
volume of thoracic chamber in the antero-
posterior axis.
87. In which of the following animals is 87. निम्नलिखित में से कौनसा जंतु उसके उत्सर्जी पदार्थ तथा
correctly matched with its waste product उत्सर्जी संरचनाओं से सही सुमेलित है:-
and excretory structures ?
(A)
(A)   जंतु    उत्सर्जी पदार्थ  उत्सर्जी संरचना
  Animal Waste product Excretory structure 1 मेंढक NH3 किडनी
1 Frog NH3 kidney
(B)
(B)   जंतु    उत्सर्जी पदार्थ  उत्सर्जी संरचना
  Animal Waste product Excretory structure 2 कें चुआ  यूरिया प्रोटोनेफ्रीडिया
2 Earthworm urea Protonephridia
(C)
(C)   जंतु    उत्सर्जी पदार्थ  उत्सर्जी संरचना
  Animal Waste product Excretory structure
3 कॉकरोच  यूरिक अम्ल मैलपिगी नलिकाऐं
3 Cockroach Uric acid Malpighian tubules

(D) (D)
  Animal Waste product Excretory structure   जंतु    उत्सर्जी पदार्थ  उत्सर्जी संरचना
4 Prawn NH3 Nephridia 4 झींगा  NH3 नेफ्रीडिया

88. Find out the correct statement from the 88. निम्न में से सही कथन पहचानिये: 
following.
से अंतः क्रिया करते
(a) हार्मोन जो झिल्ली से जुडे ग्राही
(a) Hormones which interact with है और समान्य रूप से लक्ष्य कोशिका में प्रवेश नही करते
membrane bound receptors and normally है। 
do not enter the target cells (b)
आयोडोथायरॉक्सिन प्लाज्मा झिल्ली पर उपस्थित
ग्राही से जुडता है। 
(b) Iodothyroxines have plasma membrane

(c) हार्मोन जो अन्तराकोशिकीय ग्राही से अन्तः क्रिया


bound receptors.
करते है  तथा  अधिकांश जीन अभिव्यक्ति को नियंत्रित
(c) Hormones which interact with करते है। 
intracellular receptors mostly regulate (d) स्टेरॉइड
हार्मोन को द्वितीयक संदेशवाहको की
gene expression. आवश्यकता होती है। 
(d) Steroid hormones requires secondary
(A) के वल (a) तथा (b)
messengers.
(A) Only (a) and (b) (B) के वल (b) तथा (c)

(B) Only (b) and (c) (C) के वल (a) तथा (c)

(C) Only (a) and (c) (D) के वल (b) तथा (d)

(D) Only (b) and (d)


89. An intestinal parasite which causes 89. एक आंत्रीय परजीवी जो आंत्रीय मार्ग  को अवरूद्ध कर
blockage of the intestinal passage and देता है और जिसके अण्डे संक्रमित व्यक्ति के मल के
whose eggs are excreted along with the साथ बाहर निकलते है, वह है-
faeces of infected person is-
(A) वुचेरे रिया बैंक्रोफ्टाई
(A) Wuchereria bancrofti
(B) एस्के रिस
(B) Ascaris
(C) Epidermophyton (C) एपीडर्मोफायटाॅन

(D) Microsporum (D) माइक्रोस्पोरम

90. Which of the following drugs is normally 90. निम्नलिखित में से कौनसा ड्र ग अवसाद व अनिद्रा जैसे
used as medicines to help the patients मानसिक व्याधि से ग्रस्त रोगियों की सहायता के लिए
cope with mental illness like depression सामान्यतः औषधियों के रूप में काम मे ली जाती है?
and insomnia and are often abused?
(A) एफे टीमीन
(A) Amphetamines
(B) बार्बिट्यूरे ट
(B) Barbiturates
(C) Benzodiazepines (C) बेंजोडायजेपीन

(D) All of these (D) इनमें से सभी

91. Read the following statements and choose 91. निम्नलिखित कथनों को पढ़े और असत्य कथन चुने-
which are not correct -  (a) प्लाज्मा में   90 – 92%  जल व 6 - 8%  प्रोटीन
(a) Plasma has 90 – 92% water & 6 – 8% होते हैं
proteins  (b)  इरिथ्रोसाइट को संगठित पदार्थ कहते है व ये रक्त
(b) Erythrocytes are called as formed
का 55% बनाते है
elements & constitute 55% of blood
(c) Granulocytes include basophil, (c)  ग्रैन्यूलोसाइट    में बेसोफिल, न्यूट्रोफिल और
neutrophil & monocyte  मोनोसाइट शामिल होते हैं
(d) Reduction in number of platelets may (d)  प्लेटलेट्स के संख्या में कमी के कारण रक्त का
cause excessive blood loss  अत्यधिक ह्यास होता है
(A) a and b (A) a और b
(B) b and c (B) b और c
(C) c and d (C) c और d
(D) b, c and d (D) b, c और d
92. Given below are two statements : 92. नीचे दो कथन दिए गए हैं -
Statement I : Annelids, arthropods and कथन I : ऐनेलिडा, आर्थ्रोपोडा तथा मोलस्का खण्डीत
molluscs are segmented coelomates. गुहीय जंतु है। 
Statement II : Coelentrates, ctenophores कथन II :  सिलेन्ट्रेटा, टीनोफोरा तथा इकाईनोडर्म में
and echinoderms have radial symmetry. अरीय सममिति पाई जाती है।

In the light of the above statements उपरोक्त कथनों पर प्रकाश डालिये तथा निचे दिये गये
choose the most appropriate answer from विकल्पों से उपयुक्त उत्तर का चयन करे :
the options given below :
(A) दोनो कथन I तथा कथन II गलत है
(A) Both statement I and statement II are
incorrect. (B) कथन I सही है लेकिन कथन II गलत है
(B) Statement I is correct but statement II (C) कथन I गलत है लेकिन कथन II सही है
is incorrect.
(D) दोनो कथन I तथा कथन II सही है
(C) Statement I is incorrect but statement
II is correct.
(D) Both statement I and statement II are
correct.
93. Main function of eustachian tube is to :- 93. यूस्टेकियन ट्यूब का मुख्य कार्य है :-
(A) Protect tympanic membrane from (A)  टिम्पैनिक कला की जीवाणुओं से सुरक्षा
bacteria
(B) मध्य कर्ण की अस्थियों को सहारा देना
(B) Support the bones of middle ear
(C) टिम्पैनिक कला के दोनों ओर दाब को बराबर करना
(C) Equalize pressure on two sides of
tympanic membrane (D) कर्ण पटल में संक्रमण को रोकना
(D) Prevent infection entering ear drum
94. Which organ receives only oxygenated 94. मानव में कौनसा अंग के वल ऑक्सीजनित रक्त प्राप्त
blood in human ? करता है
(A) Heart (A) हृदय
(B) Kidney (B) वृक्क
(C) Lung
(C) फें फड़े 
(D) Liver
(D) यकृ त

95. Which is not included in the female 95. निम्न में से किसे मादा बाह्य जननेन्द्रियों (female
external genitalia? external genitalia) में सम्मिलित नहीं करते है?
(A) Clitoris (A) भगशेफ (clitoris) को
(B) Mons pubis (B) जघन शैल (mons pubis) को
(C) Vagina
(C) योनि (vagina) को
(D) Labia majora
(D) वृहद् भगोष्ठ (labia majora) को

96. Which kind of uterine changes occur during 96. मासिक चक्र की पुटकिय प्रावस्था के दौरान गर्भाश्य में
follicular phase of menstrual cycle ? किस प्रकार का परिवर्तन होता है ? 
(A) Degeneration of endometrium  (A) एं डोमेट्रि यम (गर्भाशय अन्तः स्तर) का अधः पतन
(B) Regeneration of endometrium through (degeneration)
proliferation  (B) प्रचुरोद्धभवन् के माध्यम से एं डोमेट्रि यम (गर्भाशय
(C) Secretory changes of endometrium for अन्तः स्तर) का पुनरूद्धभवन 
pregnancy 
(C) गर्भावस्था के लिए एं डोमेट्रि यम (गर्भाशय अन्तः स्तर)
(D) Contraction in myometrium  का स्त्रावी परिवर्तन 

(D) मायोमेट्रि यम (गर्भाशय पेशी स्तर) में संकु चन

97. Middle lamella is composed mainly of ? 97. मध्य पटलिका मुख्यतः किसकी बनी होती है :
(A) Hemicellulose (A) हेमीसेलुलोज
(B) Calcium pectate (B) कै ल्शियम पेक्टेट
(C) Muramic acid (C) मुरै मिक अम्ल
(D) Phosphoglycerides
(D)  फॉस्फोग्लिसराइड्स
98. Identify the wrong statement about 98. अर्द्धसूत्री विभाजन के बारे में गलत कथन की पहचान
meiosis कीजिए-
(A) Pairing of homologous chromosomes (A) समजात गुणसूत्रो का युग्मन
(B) Four haploid cells are formed (B) चार अगुणित कोशिकाएँ बनती है।
(C) At the end of meiosis the number of
(C) अर्द्धसूत्री विभाजन के अंत में गुणसूत्रो की संख्या
chromosomes are reduced to half.
आधी रह जाती है।
(D) Two cycle of DNA replication occurs
(D) DNA प्रतिकृ तियन के दो चक्र होते है। ​
99. Select the true statements from the 99. निम्नलिखित में से सही कथनों का चयन कीजिए-
following: (A) जन्तुओं से प्राप्त इन्सुलीन से कु छ रोगियों में एलर्जी
(A) Insulin from animal source, may हो सकती है
develop allergy in some patients. (B) परिपक्व इन्सुलीन में सी-पेप्टाईड उपस्थित नहीं होता
(B) C-peptide is not present in mature है
insulin (C) पुनर्योगज चिकित्सा अवांछित प्रतिरक्षा अनुक्रिया को
(C) Recombinant therapeutics do not प्रेरित नही करती है
induce unwanted immunological response. (D) डायबिटीज के रोगी को इन्सुलीन मुख से दी जा
(D) Insulin can be administered orally to सकती है।
diabetic patients.
(A) के वल A व C
(A) A and C only
(B) A and B only (B) के वल A व B

(C) C and D only (C) के वल C व D

(D) A, B and C only (D) के वल A, B व C

100. After  5 cycles of PCR how many DNA 100. PCR  के 5 चरण के पश्चात् दिये गये DNA से कितने
fragments are formed from a given DNA : DNA खण्ड बनेगे।
(A) 32
(A) 32
(B) 5
(B) 5
(C) 10
(C) 10
(D) 16
(D) 16
Physics

101. The pressure at the bottom of a tank 101. एक द्रव को रखने वाले एक टैंक की तली पर दाब निर्भर
containing a liquid does not depend on नहीं करता है-
(A) Acceleration due to gravity (A) गुरुत्वीय त्वरण पर 
(B) Height of the liquid column (B) द्रव स्तम्भ की ऊॅ चाई पर
(C) Area of the bottom surface
(C) तली की सतह के क्षेत्रफल पर 
(D) Nature of the liquid
(D) द्रव की प्रकृ ति पर
102. Given that v is the speed, r is radius and g 102. यदि v चाल, r त्रिज्या तथा g गुरूत्वीय त्वरण है, तब
is acceleration due to gravity. Which of the इनमें से कौन विमाहीन है
following is dimension less 2

(A)
v g
2

(A)
v g r

(B) v2rg
(B) v2rg
(C) vr2g
(C) vr2g
(D)
2
v

(D) v
2 rg

rg

103. De-Broglie wavelength depends upon mass 103. डी ब्रोग्लि तरं गदैर्ध्य द्रव्यमान 'm' तथा गतिज ऊर्जा  'E'
'm' and kinetic energy 'E' according to the पर निम संबंध के अनुसार निर्भर करती है
relation as-
(A) mE−1/2
(A) mE−1/2
(B) m1/2E
(B) m1/2E
(C) m−1/2E−1/2
(C) m−1/2E−1/2
(D) m−1/2E
(D) m−1/2E
104. A copper ring is held horizontally and a bar 104. एक तांबे की वलय क्षेतिज रूप से रखी जाती है तथा एक
magnet is dropped through the ring with छड चुम्बक वलय के अक्ष के अनुदिश इसकी लम्बाई के
its length along the axis of the ring. The साथ वलय के माध्यम से गिराई जाती है। गिरती हुई
acceleration of the falling magnet while it
चुम्बक का त्वरण जब यह वलय से गुजर रही है, है-
is passing through the ring is
(A) गुरूत्वीय त्वरण के बराबर
(A) Equal to that due to gravity
(B) Less than that due to gravity (B) गुरूत्वीय त्वरण से कम

(C) More than that due to gravity (C) गुरूत्वीय त्वरण से अधिक

(D) Depends on the diameter of the ring (D) चुम्बक की लम्बाई तथा वलय के व्यास पर निर्भर
and the length of the magnet
105. Four particles, each of mass M and 105. प्रत्येक द्रव्यमान  M के चार कण जो कि एक दू सरे से
equidistant from each other, move along a समान दू री पर हैं, एक दू सरे के अन्योन्य गुरूत्वाकर्षण
circle of radius R under the action of their प्रभाव में त्रिज्या  R के एक  वृत्त पर गतिशील है। प्रत्येक
mutual gravitational attraction. The speed कण की चाल है :
of each particle is :
−−−−−−−−−−−

−−−−−−−−−−−
– (A) √
GM
(1 + 2√2)
(A) √
GM
(1 + 2√2)
R
R

−−−−−−−−−−−
−−−−−−−−−−− (B) 1

GM –
(1 + 2√2)
(B)
 
1 GM – 2
√ R
(1 + 2√2)
2 R
−−−
−−− (C) √
GM

(C) √
GM

R
R

−−−−−−
−−−−−− (D) – GM
√ 2√2
(D) – GM
√ 2√2
R
R
106. Two spherical bodies P (radius 9 cm) and Q 106. दो गोलाकार पिंड P (त्रिज्या 9 सेमी) और Q (त्रिज्या 27
(radius 27 cm) are at temperature TP and सेमी) क्रमशः  TP  और TQ तापमान पर हैं। यदि P और
TQ respectively. If the maximum intensities Q के उत्सर्जन स्पेक्ट्रा में अधिकतम तीव्रता क्रमशः 300
in the emission spectra of P and Q are , nm और 900 nm पर है  तो P  द्वारा उत्सर्जित ऊर्जा
respectively, at 300 nm and 900 nm, what
की दर से Q द्वारा उत्सर्जित ऊर्जा की दर का अनुपात
is the ratio of the rate of energy radiated
by P to that by Q ? क्या है?

(A) 6 (A) 6

(B) 7 (B) 7

(C) 8 (C) 8

(D) 9 (D) 9

107. A liquid flows in a tube from left to right as 107. एक द्रव एक नली में चित्रानुसार बाये से दाये
shown in figure. A1 and A2 are the cross- प्रवाहित होती है। A1 तथा A2 दिखायेनुसार नली के भागों
section of the portions of the tube as के अनुप्रस्थ काट है। तब चालों का अनुपात v1  /
shown. Then the ratio of speeds v1  / v2  v2 होगा
will be

(A) A1 / A2
(A) A1 / A2
(B) A2 / A1
(B) A2 / A1

−− −
−−

−− −
−− (C) √A2 /√A1
(C) √A2 /√A1

−− −
−−

−− −
−− (D) √A1 /√A2
(D) √A1 /√A2

108. A conducting rod PQ  of length L = 1. 0m 108. लम्बाई L = 1. 0m की एक चालक छड़ PQ  कागज के
m  is moving with a uniform speed  भीतर निर्देशित एक एकसमान चुम्बकीय क्षेत्र
v = 2 m/ sec   in a uniform magnetic field
B = 4. 0 T  में एक समान चाल   v = 2 m/ sec  के साथ
B = 4. 0 T directed into the paper. गतिमान है। धारिता  C = 10μF का एक संधारित्र
A  capacitor of capacity C = 10μF is
चित्रानुसार  जोड़ा जाता है तो 
connected as shown in figure. Then

(A) qA = +80μC  and  qB = −80μC


(A) qA = +80μC  and  qB = −80μC

(B) qA = −80μC  and  qB = +80μC


(B) qA = −80μC  and  qB = +80μC

(C) qA = 0 = qB
(C) qA = 0 = qB

(D) संधारित्र में संग्रहित आवेश समय के साथ


(D) Charge stored in the capacitor
increases exponentially with time चरघातांकीय रूप से बढता है
109. Match the following:- 109. निम्न का मिलान कीजिए:-
Column I Column II Column I Column II
Distance A वस्तु द्वारा तय दू री P  शून्य त्वरण

A travelled B एक समान वेग


1
Q ut +
2
at
P Zero acceleration 2

by body C स्पिडो मीटर R तात्क्षणिक चाल


2

D ऊर्ध्वाधर फें की गई वस्तु की ऊँ चाई


u
1 S
B Uniform velocity Q ut +
2
at 2g
2

Instantaneous
C Speedometer R
speed (A) A-Q, B-P, C-R, D-S
Height of vertical 2

(B) A-P, B-Q, C-S, D-R


u
D S
thrown body 2g

(A) A-Q, B-P, C-R, D-S (C) A-S, B-R, C-P, D-Q

(B) A-P, B-Q, C-S, D-R (D) A-S, B-R, C-Q, D-P

(C) A-S, B-R, C-P, D-Q


(D) A-S, B-R, C-Q, D-P
110. The variation of acceleration due to gravity 110. पृथ्वी के के न्द्र से दू री d के साथ गुरुत्वीय त्वरण g की
g with distance d from centre of the earth भिन्नता को निम्न में से किस ग्राफ में सबसे सही दर्शाया
is best represented by (R = Earth's radius) गया है? (R= पृथ्वी  की त्रिज्या):
:

(A)
(A)

(B)
(B)

(C)
(C)  

(D)
(D)

111. Which one of the following is  ν – T  graph m 111. पूर्ण कृ ष्णिका के लिए निम्नलिखित में से कौनसा
for perfectly black body ?  ν is the m ν – T ग्राफ है?  ν अधिकतम तीव्रता वाले विकिरण की
m m

frequency of radiation with maximum आवृत्ति है। T परम तापमान है


intensity. T is the absolute temperature

(A) A
(A) A
(B) B
(B) B
(C) C
(C) C
(D) D
(D) D
112. The radius of 64 64
29Cu  nucleus will be 112. 29Cu नाभिक की त्रिज्या होगी-
(A) 1.2 × 10-15 m (A) 1.2 × 10-15 m
(B) -2.4 × 10-15 m (B) -2.4 × 10-15 m
(C) 3.6 × 10-15 m (C) 3.6 × 10-15 m
(D) 4.8 × 10-15 m (D) 4.8 × 10-15 m
113. At the mid point of a line joining an 113. एक इलेक्ट्रॉन और एक प्रोटॉन को मिलाने वाली रे खा के
electron and a proton, the values of  E and मध्य बिंदु पर E और V के मान होगे 
V will be.
(A) E = 0, V ≠ 0
(A) E = 0 , V≠0
(B) E ≠ 0, V = 0
(B) E ≠ 0 , V = 0
(C) E ≠ 0, V ≠ 0
(C) E ≠ 0 , V ≠ 0
(D) E = 0, V = 0
(D) E = 0 , V = 0
114. A train of mass  M is moving on a circular 114. एक ट्रेन जिसका द्रव्यमान M है एक नियत चाल   V से
track of radius 'R ' with constant speed V. वृत्ताकार पथ जिसकी त्रिज्या 'R ' है मे गति कर रही है।
The length of the train is half of the ट्रेन की लम्बाई ट्रेक की परिधि की आधी है। ट्रेन का
perimeter of the track. The l inear रे खीय संवेग होगा
momentum of the train will be
(A) शून्य
(A) 0

(B) 2 MV

(B) 2 MV

π
π

(C) MVR
(C) MVR

(D) MV ​
(D) MV

115. The U nucleus has a binding energy of


238
115. 238
नाभिक की बन्धन ऊर्जा प्रति न्यूक्लिऑन लगभग
U

about 7. 6 MeV per nucleon. If the nucleus 7. 6 MeV है। यदि नाभिक दो समान भागों
were to fission into two equal fragments, (fragments) में विखण्डीत हुआ, प्रत्येक की गतिज
each would have a kinetic energy of just ऊर्जा ठीक 100 MeV के ऊपर है। इससे, यह निष्कर्ष
over 100 MeV. From this, it can be
निकाला जा सकता है, कि–
concluded that
(A) nuclei near A = 119 must be bound by (A) A = 119 के समीप नाभिक का द्रव्यमान
about 8.44MeV / Nucleon लगभग 8.44MeV / न्यूक्लिओन से सीमित होना चाहिए

(B) nuclei near A = 119 have masses (B) A = 119 के समीप नाभिक  238


 के आधे द्रव्यमान
U

greater than half that of  U 238


से ज्यादा द्रव्यमान रखता है।
(C) nuclei near A = 119 must be bound by (C) A = 119 के समीप नाभिक लगभग 6. 7 MeV /

about 6. 7 MeV / nucleon न्यूक्लिओन से सीमित होना चाहिए


(D) None of these (D) इनमें से कोई नहीं
116. Potential energy of a bar magnet of 116. चुम्बकीय आघूर्ण M वाली दण्ड चुम्बक, जिसे B प्रेरण
magnetic moment M placed in a magnetic वाले चुम्बकीय क्षेत्र में रखा जाता है एवं यह B की दिशा
field of induction B such that it makes an के साथ θ कोण बनाती है, की स्थितिज ऊर्जा है -
angle θ with the direction of B is -
(A) MB sin θ
(A) MB sin θ

(B) – MB cos θ
(B) – MB cos θ

(C) MB (1 - cos θ)
(C) MB (1 - cos θ)
(D) MB (1+ cosθ)
(D) MB (1+ cosθ)
117. A planet revolves around the sun in an 117. एक ग्रह सूर्य के चारों ओर दीर्घ वृत्ताकार पथ पर घूर्णन
elliptical orbit. If vp and va are the कर रहा है। यदि  v और  v क्रमशः उपभू व अपभू पर
p a

velocities of the planet at the perigee and ग्रह के वेग है, तो दीर्घवृत्तीय कक्षा की उत्के न्द्रता होगी -
apogee respectively, then the eccentricity
of the elliptical orbit is given by- (A)
Vp

Va

(A)
Vp

(B)
Va −Vp
Va
Va +Vp

(B)
Va −Vp

(C)
Vp +Va
Va +Vp
Vp −Va

(C)
Vp +Va

(D)
Vp −Va
Vp −Va
Vp +Va

(D)
Vp −Va

Vp +Va

118. When heat energy of 1500 J is supplied to 118. जब स्थिर दाब 2.1 × 105 N m–2 पर 1500 J  की
a gas at constant pressure, 2.1 × 105 N ऊष्मा ऊर्जा की आपूर्ति की जाती है तो इसके आयतन में
m–2, there was an increase in its volume 2.5 × 10–3  m3  के बराबर वृद्धि होती है। इसकी
equal to 2.5 × 10–3 m3. The increase in its
आंतरिक ऊर्जा में वृद्धि  है
internal energy is
(A) 450 J
(A) 450 J
(B) 525 J
(B) 525 J
(C) 975 J
(C) 975 J
(D) 2025 J
(D) 2025 J
119. Figure shows two equipotential lines in  XY 119. चित्रानुसार एक विद्युत क्षेत्र के लिए  X − Y तल में दो
plane for an electric field. The scales are समविभव रे खायें है। पैमाना चिन्हित किया है। इन
marked. The X−component E     and x
समविभव रे खाओं के बीच स्थान में विद्युत क्षेत्र के
Y−component E     of the electric field in
E X-घटक और E Y-घटक क्रमशः है -
y
x y
the space between these equipotential
lines are respectively.

(A) +100 Vm
−1
, −200 Vm
−1

(A) +100 Vm
−1
, −200 Vm
−1

(B) +200 Vm
−1
, +100 Vm
−1

(B) +200 Vm
−1
, +100 Vm
−1

(C) −100 Vm
−1
, +200 Vm
−1

(C) −100 Vm
−1
, +200 Vm
−1

(D) −200 Vm
−1
, −100 Vm
−1

(D) −200 Vm
−1
, −100 Vm
−1

120. A particle is on a smooth horizontal plane. 120. एक कण किसी चिकने क्षैतिज तल पर है। एक बल  F
A force F is applied whose F-t graph is लगाया गया है जिसका F-t  आरे ख दिया गया है तो-
given. Then :

(A) t1  पर त्वरण नियत होता है 


(A) at t1  acceleration is constant
(B) अन्त में त्वरण शून्य होता है 
(B) Finally acceleration is zero
(C) t2  पर त्वरण नियत होता है 
(C) at t acceleration is constant
2

(D) None of these (D) इनमे से कोई नहीं


121. In the given branch AB of a circuit a 121. किसी परिपथ की दी गयी शाखा AB में एक धारा
current I = (10t + 5)A is flowing, where t is I = (10t + 5)A प्रवाहित हो रही है, जहाँ  t सेकण्ड में

time in second. At t = 0, the potential समय है। t = 0  पर ,बिन्दुओ A  तथा  B    के मध्य


difference between points A and विभवान्तर (V − V )  होगा
− V ) is
A B
B (V A B

(A) 15 V
(A) 15 V

(B) −5 V
(B) −5 V

(C) −15 V
(C) −15 V

(D) 5 V
(D) 5 V

122. Which statement is correct for p-type 122. P- प्रकार  के अर्धचालक के लिए कौनसा कथन सही है -
semiconductor-
(A) सुचालन बैण्ड  में इलेक्ट्रॉन्स की संख्या कक्षीय  ताप
(A) the number of electrons in conduction पर संयोजी बैण्ड में हॉल्स  की संख्या से अधिक होती है
band is more than the number of holes in
valence band at room temperature (B) संयोजी बैण्ड में हॉल्स  की संख्या  कक्षीय तापमान
पर चालन बैण्ड  में इलेक्ट्रॉन्स की संख्या से अधिक होती
(B) the number of holes in valence band is
more than the number of electrons in है |
conduction band at room temperature (C) कक्षीय ताप पर इलेक्ट्रॉन्स तथा  कोई हॉल्स  नहीं
(C) there are no holes and electrons at होता है |
room temperature
(D) हॉल्स  तथा इलेक्ट्रॉन्स की संख्या संयोजी तथा चालन
(D) number of holes and electrons is equal बैण्ड में बराबर होती है |
in valence and conduction band
123. A simple pendulum performs simple 123. एक सरल लोलक X = 0 के परितः   आयाम 4  cm और
harmonic motion about X = 0 with an आवर्तकाल   2  sec के साथ सरल आवर्त गति करता है। 
amplitude 4  cm and time period 2  sec. The X = A/2 पर  लोलक की चाल ( cm/s में) होगी
speed of the pendulum at X = A/2 will be

(in cm/s): (A) 2√3π

(A)

2√3π (B) 2π


(B) 2π (C) √3π

(C)

√3π (D) 6π
2

(D) 6π
2

124. Assuming the expression for the pressure 124. गैस द्वारा पात्र की दीवारों पर लगाए गए दाब के व्यंजक
exerted by the gas on the walls of the को मानकर यह दिखाया जा सकता है कि दाब है
container, it can be shown that pressure is
(A) [
1
]  किसी गैस की प्रति इकाई आयतन गतिज ऊर्जा
(A) [ kinetic energy per unit volume of
rd 3
1
]
3

a gas (B) [
2

3
]  किसी गैस की प्रति इकाई आयतन गतिज ऊर्जा

(B) [  kinetic energy per unit volume of (C) किसी गैस की प्रति इकाई आयतन गतिज ऊर्जा
2 rd 3
] [ ]
3 4

a gas
(D) 3

2
× किसी गैस की प्रति इकाई आयतन गतिज ऊर्जा
(C) [ kinetic energy per unit volume of
rd
3
]
4

a gas
(D) 3

2
× kinetic energy per unit volume of a
gas
125. Which of the following represents the 125. एक द्विध्रुव की समविभव रे खाएं किसमें प्रदर्शित है ?
equipotential lines of a dipole ?

(A)
(A)

(B)
(B)

(C)
(C)

(D)
(D)

126. A cube of edge a has its edges parallel to 126. भुजा a के एक घन की भुजाये आयताकार निर्देशांक
x, y and z-axis of rectangular coordinate प्रणाली के   x , y  और z- अक्ष के समान्तर  हैं। एक


system. A uniform electric field E is parallel समान विद्युत क्षेत्र E , y-अक्ष के समान्तर  है और एक

to y-axis and a uniform magnetic field  B is →
समान चुंबकीय क्षेत्र B , x- अक्ष के समान्तर है। वह दर
parallel to x-axis. The rate at which energy जिस पर घन के प्रत्येक फलक से ऊर्जा प्रवाहित होती है
flows through each face of the cube is:

(A) , x-y तल के समान्तर तथा अन्य में शून्य है 


 
2
a EB

(A)  parallel to x-y plane and zero in


2
a EB 2μ 0

0

others (B) , x-y तल के समान्तर तथा अन्य में शून्य है


2
a EB

2μ 0

(B)  parallel to x-y plane and zero in


2
a EB

(C)  सभी फलको से 


2
a EB
2μ 0
2μ 0
other
(D) , y-z फलको के समान्तर तथा अन्य में शून्य
2
a EB

(C)  from all faces


2
a EB 2μ 0
2μ 0

(D)  parallel to y-z faces and zero in


2
a EB

2μ 0

others.
127. An unbanked curve has a radius of 60 m. 127. एक अबंकित वक्र की त्रिज्या 60m है। वह अधिकतम
The maximum speed at which a car can चाल जिस पर एक कार एक घुमाव बना सकती है, यदि
make a turn if the coefficient of static स्थैतिक घर्षण गुणांक 0.75 है, होगी-
friction is 0.75, is
(A) 2.1 m/s
(A) 2.1 m/s
(B) 14 m/s
(B) 14 m/s
(C) 21 m/s
(C) 21 m/s
(D) 7 m/s
(D) 7 m/s
128. Coefficient of superfical areal expansion of 128. एक ठोस का क्षेत्रीय प्रसार गुणांक 2 × 10
−5
/°C है।
a solid is 2 × 10–5 /ºC. Its coefficient of इसका रे खीय प्रसार गुणांक होगा।
linear expansion is:
(A) 1 × 10–5 /ºC
(A) 1 × 10–5 /ºC
(B) 2 × 10–5 /ºC
(B) 2 × 10–5 /ºC
(C) 3 × 10–5 /ºC
(C) 3 × 10–5 /ºC
(D) 4 × 10–5 /ºC
(D) 4 × 10–5 /ºC
129. A bob attached to a string is held  129. एक डोरी से जुड़े हुए गोलक  को क्षैतिज में रखा गया है
horizontal and released. The tension  and तथा उसको छोड़ा गया है। तनाव तथा लटकन बिन्दु से
vertical distance from point of suspension ऊर्ध्वाधर दू री के बीच वक्र होगा-
can be represented by.

(A)
(A)

(B)
(B)

(C)
(C)

(D)
(D)

130. The variation of current in a material is as 130. एक पदार्थ में धारा का परिवर्तन दर्शाया गया है, तब
shown गणना कीजिए। 

then calculate average current flow in first पहले 3 sec में औसत धारा प्रवाह है 
3 sec.
(A) 7 A
(A) 7 A
(B) 6 A
(B) 6 A
(C) 5 A
(C) 5 A
(D) 4 A
(D) 4 A
131. A body A of mass M while falling vertically 131. M द्रव्यमान की एक वस्तु A जब गुरूत्व के अन्तर्गत
downwards under gravity breaks into two उर्ध्वाधर नीचे की ओर गिरती है तब यह दो भागो में टू ट
parts; a body B of mass M and, a body C
1

3 जाती है। B भाग का द्रव्यमान    M तथा C भाग का


1

3
of mass M. The center of mass of bodies
2

3 द्रव्यमान  M होता है। वस्तु A की तुलना में वस्तुओं  B


2

B and C taken together shifts compared to


3

तथा C (एक साथ लेते हुए) का द्रव्यमान के न्द्र विस्थापित


that of body A towards:
होगा -
(A) depends on height of breaking
(A) द्रव्यमान की टू टने की ऊचाई पर निर्भर करता है 
(B) does not shift
(B) विस्थापित नही होगा 
(C)  body C
(C)  वस्तु C की ओर 
(D) body B
(D)  वस्तु B की ओर 
132. How does the angle of minimum deviation 132. यदि आपतित बैंगनी प्रकाश को लाल प्रकाश से
of glass prism vary, if the incident  violet परिवर्तित  कर  दिया जाए  तो काँच के प्रिज्म के न्यूनतम
light is replaced by red light विचलन का कोण किस प्रकार परिवर्तित होता है
(A) Decreases (A) घटता है
(B) Increases
(B)  बढ़ता है
(C) Constant
(C) स्थिर
(D) None of these
(D) इनमें से कोई नहीं
133. If the focal length of objective lens is 133. यदि, अभिदृश्यक लैंस की फोकस दू री को बढ़ा दिया-
increased then magnifying power of
(A) सूक्ष्मदर्शी की आवर्धन क्षमता बढ़ जाएगी तथा
(A) microscope will increase but that of दू रदर्शक की कम हो जाएगी।
telescope decrease
(B) सूक्ष्मदर्शी तथा दू रदर्शक दोनों की आवर्धन क्षमता
(B) microscope and telescope both will
बढ़ जाएगी।
increase
(C) microscope and telescope both will (C) सूक्ष्मदर्शी तथा दू रदर्शक दोनों की आवर्धन क्षमता
decrease कम हो जाएगी।

(D) microscope will decrease but that of (D) सूक्ष्मदर्शी की आवर्धन क्षमता कम होगी तथा
telescope increase. दू रदर्शक की बढ़ जाएगी।
134. Young's experiment proves which of 134. यंग का प्रयोग प्रमाणित करता है कि-
following fact :
(A) प्रकाश कणों से बना होता हैॅ 
(A) light is made up of particles
(B) प्रकाश तरं गों से बना होता है
(B) light is made up of waves
(C) प्रकाश न तरं गों से और न कणों से बना होता है
(C) light is made up of neither waves nor
particles (D) फ्रिज की चौड़ाई दोनों स्लिटों के बीच की दू री पर
(D) fringe width doesn't depend upon the निर्भर नहीं करती है। 
spacing between slits
135. The point of minimum intensity in 135. व्यतिकरण में न्यूनतम तीव्रता का बिंदु पूर्ण रूप से अदीप्त
interference are perfectly dark while the (dark) है जबकि विवर्तन प्रतिरूप में न्यूनतम तीव्रता का
point of minimum intensity in diffraction बिंदु:
pattern are :
(A) भी पूर्ण रूप से अदीप्त होता है।
(A) also perfectly dark
(B) एकसमान तीव्रता का होता है।
(B) of uniform intensity
(C) not perfectly dark (C) पूर्ण रूप से अदीप्त नहीं होता है।

(D) none of the above (D) इनमे से कोई नहीं।


Physics

136. If the voltage between the terminals A and 136. यदि टर्मिनलों A और B के बीच वोल्टेज 17 V है और
B is 17 V and Zener breakdown voltage is जेनर  भंजन वोल्टता  9 V है, तो R के सिरों पर विभव
9 V, then the potential across R is  है

(A) 6 V (A) 6 V
(B) 8 V (B) 8 V
(C) 9 V (C) 9 V
(D) 17 V (D) 17 V
137. A string fixed at both ends. A vibrating fork 137. एक तार दोनों सिरों पर स्थिर है। एक स्वरित्र द्विभुज  इसे
divides it into 6 loops when tension in 6 लूपों में विभाजित करता है,जब तार में तनाव 36 N है।
string is 36 N. The tension at which it will जिस तनाव पर यह 4 लूपों में कं पन करे गा, वह है
vibrate in 4 loops is.
(A) 24 N
(A) 24 N

(B) 36 N
(B) 36 N

(C) 64 N
(C) 64 N

(D) 81 N
(D) 81 N

138. The average degree of freedom per 138. एक गैस के लिए प्रति अणु स्वतंत्रता की औसत कोटि 6
molecule for a gas is 6. The gas performs है। स्थिर दाब पर प्रसार पर गैस  25 J कार्य करती है।
25 J of work when it expands at constant गैस द्वारा अवशोषित ऊष्मा है
pressure. The heat absorbed by the gas is
(A) 75 J
(A) 75 J
(B) 100 J
(B) 100 J
(C) 150 J
(C) 150 J
(D) 125 J
(D) 125 J
139. For electron of energy E, wavelength will 139. E ऊर्जा के इलेक्ट्रॉन के लिए तंरगदैर्ध्य होगी:
be : −−−−
(A) λ√2 mE
−−−−
(A) λ√2 mE
−−−−
(B) 2h/√2 mE
−−−−
(B) 2h/√2 mE
−−−−
(C) h/√2 mE
−−−−
(C) h/√2 mE
(D) h/2 mE

(D) h/2 mE
140. A square loop of wire, side length 10  cm is 140. भुजा की लंबाई  10  cm का तार का एक वर्गाकार लूप, 
placed at an angle of 45° with a magnetic 45° के कोण पर एक चुंबकीय क्षेत्र के साथ    रखा गया
field that changes uniformly from 0. 1 T to है  जो  0. 7  सेकं ड में समान रूप से  0. 1 T से शून्य में
zero in 0. 7 seconds. The induced current in परिवर्तित हो जाता है। लूप में प्रेरित धारा (इसका प्रतिरोध
the loop (its resistance is 1Ω ) is:
1Ω है) है
(A) 1. 0  mA
(A) 1. 0  mA

(B) 2. 5  mA
(B) 2. 5  mA

(C) 3. 5  mA
(C) 3. 5  mA

(D) 4. 0  mA
(D) 4. 0  mA

141. A cylinder of fixed capacity of  44. 8  litres 141. किसी 44. 8 लीटर की नियत क्षमता वाले बेलनाकार बर्तन
contains helium gas at standard (सिलैण्डर) में मानक ताप एवं दाब पर हीलियम गैस भरी
temperature and pressure. The amount of हुई है। सिलैण्डर के अन्दर की गैस के तापमान को 
heat needed to raise the temperature of 20°C बढ़ाने के लिए, ऊष्मा की आवश्यक मात्रा होगी :
gas in the cylinder by 20°C will be : (Given
(दिया है: गैस नियतांक R = 8. 3 JK − mol )
−1 −1

gas constant R = 8. 3 JK − mol )


−1 −1

(A) 249 J
(A) 249 J
(B) 415 J
(B) 415 J
(C) 498 J
(C) 498 J
(D) 830 J
(D) 830 J
142. If a long horizontal wire is bent as shown 142. यदि एक लम्बे क्षैतिज तार को चित्रानुसार मोड़ा जाए तथा
in the figure and current i is passed उसमें से धारा i प्रवाहित की जाए  तो वृत्ताकार भाग के
through it, then the magnitude and कें द्र पर उत्पन्न चुंबकीय क्षेत्र का परिमाण व दिशा होगी:
direction of magnetic field produced at the
centre of the circular part will be:

(A)
μ0 i
,⊗
r

(A)
μ0 i

(B)
μ0 i 1
,⊗ (1 − ), ⊗
r
2r π

(B)
μ0 i

(C)
1 μ0 i
(1 − ), ⊗ 1
2r π [1 + ], ⊙
2r π

(C)
μ0 i

(D)
1 μ0 i
[1 + ], ⊙ 1
2r π (1 − ), ⊗
r π

(D)
μ0 i 1
(1 − ), ⊗
r π

143. By stretching, The length of the resistance 143. खींचने पर, प्रतिरोध तार की लम्बाई में 10% की वृद्धि हो
wire is increased by 10%. What is the जाती है। तार के प्रतिरोध में संगत परिवर्तन क्या होगा
corresponding change in the resistance of
wire ? (A) 20%

(A) 20% (B) 25%

(B) 25% (C) 21%

(C) 21% (D) 9%

(D) 9%
144. A transformer having efficiency of 90% is 144. एक ट्रांसफॉर्मर की दक्षता 90%  है, यह 200V व 3
working on 200V and 3kW power supply. If किलोवाट की विद्युत  आपूर्ति  पर कार्य  कर रहा है।
the current in the secondary coil is 6A, the यदि,द्वितीयक कु ण्डली से 6 ऐम्पियर की धारा प्रवाहित हो
voltage across the secondary coil and the रही है, तो द्वितीयक कु ण्डली के सिरों के बीच वोल्टेज
current in the primary coil respectively are
तथा प्राथमिक कु ण्डली में विद्युत धारा का मान क्रमशः
-
होगा -
(A) 450 V, 13.5 V
(A) 450 V, 13.5 V
(B) 600 V, 15 A
(B) 600 V, 15 A
(C) 300 V, 15 A
(C) 300 V, 15 A
(D) 450 V, 15 A
(D) 450 V, 15 A
145. Three capacitors, each of value 1μF  are so 145. एक-एक माइक्रो फै रड के तीन संधारित्र इस तरह से एक
combined that the resultant capacity is दू सरे के साथ जोड़े गये है कि समूह की परिणामी धारिता
1. 5μF . Then:
1. 5 माइक्रो फै रड है तो :-

(A) All three capacitors are connected in (A) तीनों संधारित्र समान्तर क्रम में जुड़े है।
parallel
(B) तीनों संधारित्र श्रेणी क्रम में जुड़े है।
(B) All three capacitors are connected in
series (C) दो संधारित्रों के समान्तर क्रम में तीसरा संधारित्र
(C) Third capacitor is in series with a श्रेणीबद्ध है।
parallel combination of the other two. (D) दो संधारित्रों के श्रेणी क्रम में तीसरा संधारित्र
(D) Third capacitor is in parallel with a समान्तर रुप से जुड़ा है।
series combination of the other two.
146. The pressure exerted by an 146. एक अपरावर्तक पृष्ठ पर IW/m2 तीव्रता की विद्युत
electromagnetic wave of intensity IW/m2 चुम्बकीय तरं ग द्वारा लगाया गया दाब है:-
on a non- reflecting surface is :-
(A) Ic
(A) Ic
(B) Ic2
(B) Ic2
(C) I/c
(C) I/c
(D) I/c2
(D) I/c2
147. A truck travelling due north with 20 m/s 147. 20 m/s चाल से उत्तर की ओर गति करता हुआ एक
turns towards west and travels at the ट्र क पश्चिम की ओर मुड़ता है तथा उसी चाल से गति
same speed. Then the change in velocity is करता है। इस के वेग में परिवर्तन होगा
:-
(A) 40 m/s उत्तर-पश्चिम
(A) 40 m/s north-west
(B) 20 m/s उत्तर-पश्चिम
(B) 20 m/s north-west
(C) 40 m/s south-west (C) 40 m/s दक्षिण-पश्चिम

(D) 20√2 m/s दक्षिण-पश्चिम




(D) 20√2 m/s south-west
148. A 1  kg block is being pushed against a wall 148. 1  kg   ब्लॉक  पर  चित्रानुसार  F = 75N  बल द्वारा दीवार
by a force  F = 75N as shown in the figure. के  विरूद्ध धक्का लगाया जाता है । घर्षण गुणांक 0. 25 है
The coefficient of friction is 0. 25. The | ब्लॉक के त्वरण का परिमाण होगा
magnitude of acceleration of the block is
 

(A)  10 m/s 2

(A)  10 m/s 2

(B) 20 m/s
2

(B) 20 m/s
2

(C) 5 m/s
2

(C) 5 m/s
2

(D) None

(D) कोई नही


149. A body of mass 1  kg begins to move under 149. 1 kg द्रव्यमान का कोई पिण्ड किसी कालाश्रित बल
the action of a time dependent force  (समय पर निर्भर) 

ˆ 2ˆ
F = (2t i + 3t j )N यहाँ ˆ
i और

ˆ 2ˆ
F = (2t i + 3t j )N , where ˆ
i and ˆ
j are unit ˆ
j, x तथा y अक्ष के अनुदिश मात्रक सदिश हैं के अधीन
vectors along x and y axis. What power will गति आरम्भ करता है, तो समय t पर इस बल द्वारा
be developed by the force at the time t ? उत्पन्न शक्ति क्या होगी ?
(A) (2t
2 3
+ 3t )W
(A) (2t
2 3
+ 3t )W

(B) (2t
2 4
+ 4t )W
(B) (2t
2 4
+ 4t )W

(C) (2t
3 4
+ 3t )W
(C) (2t
3 4
+ 3t )W

(D) (2t
3 5
+ 3t )W
(D) (2t
3 5
+ 3t )W

150. Polarisation of light proves the - 150. प्रकाश के ध्रुवण द्वारा यह सिद्धान्त सिद्ध होता है-
(A) corpuscular nature of light (A) प्रकाश का  कणिका सिद्धान्त
(B) quantum nature of light (B) प्रकाश का  क्वाण्टम सिद्धान्त
(C) transverse wave nature of light (C) प्रकाश का अनुप्रस्थ तरं ग होना
(D) longitudinal wave nature of light
(D) प्रकाश का अनुदैर्ध्य तरं ग होना
Chemistry

151. A reversible process is associated with- 151. एक उत्क्रमणीय प्रक्रम संबंधित होता है
(A) constant entropy  (A) नियत एन्ट्रॉपी
(B) System and surrounding remain in (B) प्रक्रम के दौरान निकाय तथा परिवेश साम्य में होते
equilibrium throughout the process है
(C) Constant enthalpy
(C) नियत एन्थैल्पी
(D) Large change in heat content
(D) ऊष्मा घटक में अधिक परिवर्तन
152. The correct order of increasing electron 152. निम्न तत्वों की बढ़ती इलेक्ट्रॉन बंधुता का सही क्रम है
affinity of the following elements is :
(A) O < S < F < Cl

(A) O < S < F < Cl


(B) O < S < Cl < F

(B) O < S < Cl < F


(C) S < O < F < Cl

(C) S < O < F < Cl


(D) S < O < Cl < F

(D) S < O < Cl < F

153. Phenol is used in the manufacture of – 153. निम्न में से  किसके निर्माण में फिनॉल का उपयोग होता
(A) Bakelite हैं-

(B) Polystyrene (A) बेके लाइट

(C) Nylon (B) पॉलिस्टाइरीन

(D) PVC (C) नायलॉन

(D) PVC
154. The numerical value of `a' the vander 154. वाण्डर वॉल नियतांक `a'  का संख्यात्मक मान  किसके  
Wall's constant is maximum for : लिये अधिकतम  होगा ?
(A) NH3 (A) NH3
(B) H2 (B) H2
(C) O2 (C) O2
(D) He (D) He
155. In the extraction of copper from its 155. कॉपर का इसके सल्फाइड अयस्क से निष्कर्षण में, धातु
sulphide ore, the metal is finally obtained परिणामी रूप से क्युप्रस ऑक्साइड के अपचयन द्वारा
by the reduction of cuprous oxide with- निम्न के साथ प्राप्त की जाती है-
(A) sulphur dioxide (A) सल्फर डाई ऑक्साइड
(B) iron (II) sulphide
(B) आयरन (II) सल्फाइड
(C) carbon monoxide
(C) कार्बन मोनोक्साइड
(D) copper (I) sulphide
(D) कॉपर (I) सल्फाइड
156. Two statement in respect of drug-enzyme 156. औषध-एन्जाइम अन्योन्यक्रिया के संदर्भ में   कथन नीचे
interaction are given below दिए है।
Statement I : Action of an enzyme can be कथन I : एन्जाइम की क्रिया को रोका जा सकता है,
blocked only when inhibitor blocks the के वल जब संदमक एन्जाइम के सक्रिय स्थलों को अवरुद्ध
active site of the enzyme.
कर दें।
Statement II : An inhibitor can form a
strong covalent bond with the enzyme. कथन II : संदमक, एन्जाइम के साथ प्रबल सहसंयोजी
आबन्ध बना सकता है।
In the light of the above statements,
choose the correct answer from the नीचे दिए विकल्पों में से, उपरोक्त कथनों के लिए
options given below सर्वाधिक उचित उत्तर चुनिए।

(A) Both statement I and statement II are (A) कथन I तथा II दोनों सही हैं ।
true
(B) कथन I तथा II दोनो गलत हैं।
(B) Both statement I and statement II are
false (C) कथन I सही है परन्तु कथन II गलत है।

(C) statement I is true but statement II is (D) कथन I गलत है परन्तु कथन II सही है ।
false
(D) statement I is false but statement II is
true
157. (A) −−−
KOH
→ (B) −−−
LiAlH4

→ CH3 CH2 NHCH3  Identify 157. (A) −−−


KOH
→ (B) −−−
LiAlH4

→ CH3 CH2 NHCH3  यौगिक A


⊕ ⊕
CHCl 3 H CHCl 3 H

compound A : को पहचानिये

(A) (A)

(B) (B)

(C) (C)

(D) CH3 − CH2 − NH2 (D) CH3 − CH2 − NH2

158. Which arrangement of magnetic moment 158. डोमेन के चुम्बकीय आघूर्ण की कौनसी व्यवस्था
of domains decides ferrimagnetism : फे रीचुम्बकत्व को दर्शाती है
(A) ↑↑↑↑↑ (A) ↑↑↑↑↑
(B) ↑↓↑↓ (B) ↑↓↑↓
(C) ↑↑↑↓↓ (C) ↑↑↑↓↓
(D) None of these (D) कोई नहीं
159. Hybridised state of N in NO +
2
and NO

2
 are 159. NO
+
2
 तथा NO

2
 में N की संकरित अवस्था क्रमशः है
....and ..... respectively :
(A) sp
2
 तथा sp   2

(A) sp
2
 & sp  
2

(B) sp तथा sp
(B) sp & sp
(C) sp तथा sp 2

(C) sp & sp 2

(D) कोई नही


(D) None
160. IUPAC name of the given compound is : 160. दिये गये यौगिक का IUPAC नाम है

(A) 4-Ethyl-2-methylaniline (A) 4- ऐथिल-2-मेथिल ऐनीलीन

(B) 4-Amino-1-ethyl-3-methylbenzene (B) 4-ऐमीनो -1-ऐथिल-3- मेथिल बेन्जीन

(C) 4-Ethyl-6-methylaniline (C) 4- ऐथिल-6- मेथिल ऐनीलीन

(D) 4-Amino-1-ethyl-5-methylbenzene (D) 4-एमीनो-1-ऐथिल-5- मेथिलबेन्जीन

161. In the electrolysis of aqueous NaCl, how 161. जलीय NaCl के वैद्युत अपघटन में, 1 M सान्द्रता के
long would you pass a current of 1.0 A 1.0 L विलयन को 1 M NaOH में परिवर्तित करने के
through the cell to convert 1.0 L solution लिए सेल से 1.0 A की धारा कितने समय तक प्रवाहित
of 1 M concentration into 1 M NaOH? करनी होगी?
(A) 12.6 hr
(A) 12.6 hr
(B) 20.4 hr
(B) 20.4 hr
(C) 26.8 hr
(C) 26.8 hr
(D) 30.2 hr
(D) 30.2 hr
162. Which of the following is soluble in water ? 162. निम्न में से कौन जल में विलेय है  ?
(A) CS2 (A) CS2
(B) C2H5OH (B) C2H5OH
(C) CCl4 (C) CCl4
(D) CHCl3 (D) CHCl3
163. In the reaction  163. अभिक्रिया में

X​ and Y are : ​X और Y है 


(A) X=1–Butyne ; Y=3–Hexyne (A) X=1–ब्यूटाइन ; Y=3–हेक्साइन
(B) X=2–Butyne ; Y=3–Hexyne (B) X=2–ब्यूटाइन ; Y=3–हेक्साइन
(C) X=2–Butyne ; Y=2–Hexyne
(C) X=2–ब्यूटाइन ; Y=2–हेक्साइन
(D) X=1–Butyne ; Y=2–Hexyne
(D) X=1–ब्यूटाइन ; Y=2–हेक्साइन
164. Which of the following equilibrium will shift 164. निम्न में से कौनसा साम्य ताप में वृद्धि के कारण दांयी और
towards right side due to increase in विस्थापित होगा
temperature
(A) CO(g) + H2O(g) ⇌ CO2(g) + H2(g) Δ H
(A) CO(g) + H2O(g) ⇌ CO2(g) + H2(g) Δ H –ve
–ve
(B) 2SO2(g) + O2(g) ⇌ 2SO3(g) ΔH –ve
(B) 2SO2(g) + O2(g) ⇌ 2SO3(g) ΔH –ve
(C) H2O(g) ⇌ H2(g) + O2(g) ΔH +ve
1

(C) H2O(g) H2(g) + O2(g) ΔH +ve


1 2

2

(D) 4HCl(g) + O2(g) ⇌ 2H2O(g) + 2Cl2(g) 


(D) 4HCl(g) + O2(g) ⇌ 2H2O(g) + 2Cl2(g)  ΔH –ve
ΔH –ve
165. The CFSE for [(CoCl)6]4– complex is 165. संकु ल [(CoCl)6]4– के लिए CFSE का मान
18000cm–1. The Δ for [CoCl4]2– will be: 18000cm–1हैं। [CoCl4]2– के लिए Δ होगा:
(A) 18000 cm–1 (A) 18000 cm–1
(B) 16000 cm–1 (B) 16000 cm–1
(C) 8000 cm–1 (C) 8000 cm–1
(D) 2000 cm–1 (D) 2000 cm–1
166. 166.

compound not formed is : उपरोक्त अभिक्रिया में कौनसा यौगिक नही बनता है
(A) CH3 − CH = O (A) CH3 − CH = O

(B) CO2 (B) CO2

(C) (C)

(D) (D)

167. The pH of blood is maintained by CO and2–


3
167. शरीर व रक्त के रासायनिक संगठन में,  CO   व 2–
3

H2CO3 in the body and chemical H2CO3  रक्त का pH निर्धारित करते है। यह घटना
constituents of blood. This phenomenon is कहलाती है -
called -
(A) कोलाॅइडल   
(A) Colloidal
(B) बफर क्रिया
(B) Buffer action
(C) Acidity (C) अम्लीयता

(D) Salt balance (D) लवण सन्तुलन

168. Which of the following statement(s) is/are 168. निम्न में से कौनसा सही है
correct?
(A) Δt = 4/9Δ0

(A) Δt = 4/9Δ0

(B) लिगेण्ड लुईस क्षार है


(B) Ligands are Lewis bases
(C) सिस-प्लेटिन कैं सर के उपचार में प्रयुक्त होती है
(C) Cis-platin is used in the treatment of
cancer (D) उपरोक्त सभी
(D) All of these
169. 169.

Alkene (C) is : एल्कीन (C) है


(A) cis-2-butene (A) सिस-2-ब्यूटीन
(B) trans-2-butene (B) ट्रान्स-2-ब्यूटीन
(C) 1-butene (C) 1-ब्यूटीन
(D) ethene
(D) ऐथीन
170. The standard heat of combustion of solid 170. ठोस बोरोन के दहन की मानक ऊष्मा निम्न के बराबर
boron is equal to : होती है
(A) ΔH° ( B2 O 3 )
f (A) ΔH° ( B2 O 3 )
f

(B) 1/2ΔH
o
f
(B2 O 3 )
(B) 1/2ΔH
o
(B2 O 3 )
f

(C) 2ΔH° ( B2 O 3 )


f (C) 2ΔH° ( B2 O 3 )
f

(D) 3/2ΔH
o
f (B2 O 3 )
(D) 3/2ΔH
o
f (B2 O 3 )

171. Slag is a product of 171. धातुमल निम्न में से किसका मिश्रण होता है 
(A) Flux & coke (A) गालक तथा कोक
(B) Coke & metal oxide (B) कोक तथा धातु ऑक्साइड
(C) Flux & impurites (C) गालक तथा अशुद्धियां
(D) Metal & Flux
(D) धातु तथा गालक
172. On Bohr's stationary orbits - 172. बोहर की स्थिर कक्षाओं में -
(A) Electrons do not move (A) इलेक्ट्रॉन गति नही करतें
(B) Electrons move emitting radiations (B) इलेक्ट्रोन विकिरण उत्सर्जित करते हुए गति करते है
(C) Energy of the electron remains (C) इलेक्ट्रॉन की ऊर्जा नियत रहती है
constant
(D) इलेक्ट्रॉन का कोणीय संवेग h/2π है
(D) Angular momentum of the electron is
h/2π
173. Which of the following statement(s) is/are 173. निम्न में से कौनसा कथन सही है जब NaCl तथा
correct when a mixture of NaCl and K Cr O  के मिश्रण को धीरे से सान्द्र H SO  के साथ
2 2 7 2 4

K Cr O
2 2 is gently warmed with conc.
7
गर्म किया जाता है
H SO ?
2 4

(A) एक गहरी लाल वाष्प मुक्त होती है


(A) A deep red vapour is evolved
(B) क्रोमिल क्लोराइड निर्मित होता है
(B) Chromyl chloride is formed
(C) Chlorine gas is evolved (C) क्लोरीन गैस मुक्त होती है

(D) Both (A) & (B) (D) दोनो (A) तथा (B)

174. Which of the following compounds liberate 174. निम्न में से कौनसा यौगिक मेथेन गैस मुक्त करे गा जब
methane when treated with excess of इन्हें  शुष्क ईथर में मेथिल मेग्नेशियम आयोडाइड के
methyl magnesium iodide in dry ether ? आधिक्य के साथ उपचारित किया जाता है
(a) CH3–CH2–CH2OH (a) CH3–CH2–CH2OH
(b) H3C–CH2–C≡CH (b) H3C–CH2–C≡CH
(c) H3C–CH2–CO2H
(c) H3C–CH2–CO2H

(d) H3C–CH2–CHO (d) H3C–CH2–CHO


(A) a, b (A) a, b
(B) b, c (B) b, c
(C) a, b, c (C) a, b, c
(D) All the above (D) उपरोक्त सभी
175. The equilibrium constant in a reversible 175. दिये गये ताप पर उत्क्रमणीय अभिक्रिया में साम्य
reaction at given temperature - नियतांक -
(A) Depends on initial concentration of the (A) क्रियाकारकों की प्रारम्भिक सान्द्रता पर निर्भर करता
reactants है
(B) Depends on the concentration of the
(B) साम्य पर उत्पादों की सान्द्रता पर निर्भर करता है
products at equilibrium
(C) Does not depend on the initial (C) प्रारम्भिक सान्द्रता पर निर्भर नहीं करता है
concentrations (D) यह अभिक्रिया का अभिलक्षण नहीं है
(D) It is not characteristic of the reaction
176. Alkali metals give colour in Bunsen flame 176. क्षार धातु बुन्सन ज्वाला में रं ग देते हैं -इसका कारण है -
due to :
(A) कम विद्युतऋणता
(A) Low electronegativity
(B) बाह्यतम कक्ष में एक इलैक्ट्रॉन 
(B) One e  in outermost orbit

(C) छोटी परमाणु त्रिज्या


(C) Smaller atomic radii
(D) कम आयनन ऊर्जा
(D) Low ionisation energy
177. Anisole can be prepared by the action of 177. एनिसोल को सोडियम फीनेट पर मेथिल आयोडाइड की
methyl iodide on sodium phenate. The क्रिया द्वारा निर्मित किया जा सकता है। अभिक्रिया
reaction is called- कहलाती है-
(A) Fittig reaction (A) फिटिग अभिक्रिया 
(B) Etard reaction
(B) इटार्डअभिक्रिया
(C) Wurtz reaction
(C) वुर्ट्ज अभिक्रिया
(D) Williamson synthesis
(D) विलियम्सन संश्लेषण
178. If Raoult's law is obeyed, then vapour 178. यदि राऊल्ट के नियम का पालन होता है, तब एक
pressure of the solvent in a solution is विलयन में विलायक का वाष्प दाब निम्न के सीधे
directly proportional to: समानुपाती होता है
(A) Mole fraction of the solvent (A) विलायक के मोल भिन्न
(B) Mole fraction of the solute
(B) विलेय के मोल भिन्न
(C) Mole fraction of the solvent and solute
(C) विलायक तथा विलेय के मोल भिन्न
(D) The volume of the solution
(D) विलयन का आयतन
179. Identify the paramagnetic chemical species 179. अनुचुम्बकीय रासायनिक प्रजाति को पहचानिये
:
(A) H2 O 2

(A) H2 O 2
(B) KO2

(B) KO2
(C) Na2 O

(C) Na2 O
(D) उपरोक्त सभी
(D) All of these
180. Which of the following can be used to 180. निम्न में से कौनसा युग्म  तृतीयक-ब्यूटिल एल्कोहॉल के
prepare t-butyl alcohol लिए उपयोग लिया जा सकता है -
(A) CH3 MgBr + CH3 COCH3
(A) CH3 MgBr + CH3 COCH3

(B) C2 H5 MgBr + CH3 COCH3


(B) C2 H5 MgBr + CH3 COCH3

(C) CH3 MgBr + (CH3 ) C. OH


3 (C) CH3 MgBr + (CH3 ) C. OH
3

(D) CH3 MgBr + CH3 CH2 CHO


(D) CH3 MgBr + CH3 CH2 CHO
181. Iodine crystals are - 181. आयोडीन क्रिस्टल है -
(A) Metallic solid (A) धात्विक ठोस 
(B) Ionic solid (B) आयनिक ठोस 
(C) Molecular solid (C) आण्विक ठोस 
(D) Covalent solid
(D) सहसंयोजक ठोस
182. Which of the following species does not 182. निम्न में से कौनसी प्रजाति विद्मान नही होती है
exist ?
(A) OF6

(A) OF6
(B) BF
3−
6

(B) BF
3−
6
(C) XeH6

(C) XeH6
(D) उपरोक्त सभी
(D) All of these
183. Which of the given reaction is a redox 183. निम्न में से कौनसी दि गई अभिक्रिया रे डॉक्स
reaction. (अपचयोपचय) अभिक्रिया है
(A) Na2 O 2 + dil. H2 SO4 ⟶ Na2 SO4
(A) Na2 O 2 + dil. H2 SO4 ⟶ Na2 SO4

                                              + H2 O 2
                                              + H2 O 2

(B) NaCl(aq) + AgNO3 ⟶ AgCl(s)


(B) NaCl(aq) + AgNO3 ⟶ AgCl(s)

                                         + NaNO3
                                         + NaNO3

(C) F2 + H2 O ⟶ HF + O 2
(C) F2 + H2 O ⟶ HF + O 2

(D) NH3 + HCl ⟶ NH


+
4
Cl

(D) NH3 + HCl ⟶ NH
+
Cl

184. Milk is an example of : 184. दू ध किसका उदाहरण है :


(A) True solution (A) वास्तविक विलयन
(B) Gel (B) जैल   
(C) Suspension (C) निलम्बन   
(D) Emulsion
(D) पायस
185. Modified Arrhenius equation is 185. रूपान्तरित आर्हेनियस समीकरण है 

(A) k = ZAB e
–Ea
(A) k = ZAB e
–Ea

RT
RT

(B) k = PZAB e
–Ea
(B) k = PZAB e
–Ea

RT
RT

(C) k = PZAB e
–Ea T
(C) k = PZAB e
–Ea T

R
R

–E a
–E a
Z AB e
(D)
Z AB e
(D)
RT
RT
k = k =
P P
Chemistry

186. The most suitable method of separation of 186. o-तथा p-नाइट्रोफिनॉल के एक  1  :  1 मिश्रण को पृथक
a  1  :  1  mixture of o- and p-nitrophenol is करने के लिये सर्वाधिक उचित विधि है
:
(A) उर्ध्वपातन
(A) Sublimation
(B) वर्णलेखन
(B) Chromatography
(C) क्रिस्टलीकरण
(C) Crystallisation
(D) Steam Distillation (D) भाप आसवन

187. ' A ' and ' B ' formed in the following set of 187. अभिक्रियाओं के निम्न समूहो में बने 'A' और 'B' हैं:
reactions are:

(A)
(A)

(B)
(B)

(C)
(C)

(D)
(D)

188. Which of the following is acidic oxide ? 188. निम्न में से कौनसा अम्लीय ऑक्साइड है
(A) SiO 2 (A) SiO 2

(B) Al2 O 3 (B) Al2 O 3

(C) SnO2 (C) SnO2

(D) PbO (D) PbO


189. 189.

Electrophile involved in above reaction is : उपरोक्त अभिक्रिया में सम्मिलित इलेक्ट्रॉनस्नेही है


(A) Cl

(A) Cl

⊕ ⊕

(B) CH3 (B) CH3

(C) (C)

(D) (D)

190. BeF2 has no dipole moment because of : 190. BeF2किस कारण से कोई द्विध्रुव आघूर्ण नही रखता है ?
(A) Covalent bond (A) सहसंयोजक बंध
(B) Linear and symmetrical structure (B) रे खीय तथा सममितिय संरचना
(C) Non-linear structure (C) अरे खीय संरचना
(D) No charge displacement
(D) कोई आवेश विस्थापन नही

191. 191.

X can also be obtained by except ? X को निम्न में से कौनसी अभिक्रिया द्वारा प्राप्त नहीं किया
जा सकता है?
(i)  CrO2 Cl2

(A) −−−−−−→
(ii) H2 O (i)  CrO2 Cl2

(A) −−−−−−→
(ii) H2 O

(i)  CrO3   in   AC 2 O

(B) −−−−−−−−−−→
(ii) H2 O (i)  CrO3   in   AC 2 O

(B) −−−−−−−−−−→
(ii) H2 O

(i) 2 Cl2 , hν

(C) −−−−−→
(ii) H2 O (i) 2 Cl2 , hν

(C) −−−−−→
(ii) H2 O

KMnO 4

(D) −−−→
KMnO 4

(D) −−−→

192. The number of isomers exhibited by  192. [Cr (NH3 )


3
Cl 3 ]  के द्वारा प्रदर्शित समावयवीयों की
[Cr (NH ) Cl ] is :
3 3 3 संख्या है
(A) 2 (A) 2

(B) 3 (B) 3

(C) 4 (C) 4

(D) 5 (D) 5
193. Compare relative stability of following 193. निम्न अनुनादी संरचनाओं के संबंधित स्थायित्व की तुलना
resonantion structure : कीजिये

(A) a > b > c (A) a > b > c


(B) a > c > b (B) a > c > b
(C) b > a > c (C) b > a > c
(D) c > a > b (D) c > a > b
194. Which of the following statement is false : 194. निम्न में से कौनसा कथन गलत है :
(A) A fast reaction has a larger rate (A) एक तीव्र अभिक्रिया का दर नियतांक ज्यादा तथा
constant and short half life अर्ध आयुकाल कम होता है
(B) For a first order reaction, successive (B) प्रथम कोटि की अभिक्रिया के लिए क्रमागत
half lives are equal
अर्धआयुकाल समान होते है।
(C) For a first order reaction, the half life is
independent of concentration (C) प्रथम कोटि की अभिक्रिया के लिए अर्ध आयुकाल
सान्द्रता पर निर्भर नही करता है
(D) The half life of first order reaction is
half the time required for the reaction to (D) एक प्रथम कोटि अभिक्रिया का अर्ध आयुकाल
go to completion अभिक्रिया को पूर्ण होने में लगने वाले समय का आधा
होता है
195. The metallurgical process in which a metal 195. वह धातुकर्मीय विधि, जिसमें धातु संगलित अवस्था में
is obtained in a fused state is called:- प्राप्त होती है, कहलाती है:-
(A) Smelting (A) प्रगलन
(B) Roasting (B) भर्जन
(C) Calcination
(C) निस्तापन
(D) Froth flotation
(D) झाग प्लवन
196. The species given below that does NOT 196. नीचे दि गई स्पीशीज में से कौनसी विषमानुपातन
show disproportionation reaction is- अभिक्रिया नहीं दर्शाती हैं-
(A) BrO

2 (A) BrO

2

(B) BrO

4 (B) BrO

4

(C) BrO

(C) BrO

(D) BrO

3 (D) BrO

3
197. Given below are two statements: 197. नीचे दो कथन दिए है।
Statement-I: Hyperconjugation is a कथन-I अतिसंयुग्मन एक स्थायी प्रभाव है।
permanent effect.
+

Statement-II: Hyper conjugation in ethyl कथन-II  एथिल धनायन (CH3 − CH2 )   के


+

cation (CH 3 − CH2 ) involves the over अतिसंयुग्मन में C − H आबन्ध का दू सरे कार्बन के
sp
2
1s

रिक्त 2P कक्षक से अतिव्यापन होता है।


lapping of C − H  bond with empty 2p
2
sp 1s

orbital of other carbon. सही विकल्प चुनिएः


Choose the correct option: (A) दोनों कथन-I तथा कथन-II सत्य हैं।
(A) Both statement I and statement II are (B) कथन-I असत्य  है परन्तु कथन-II सत्य है।
true
(C) कथन-I सत्य है परन्तु कथन-II असत्य है।
(B) Statement I is incorrect but statement
II is true (D) दोनों कथन-I तथा कथन-II असत्य है।
(C)
Statement I is correct but statement II
is false
(D) Both Statement I and statement II are
false.


198. In the variation of ∧m  with ,
√C 198. −

√C के साथ ∧m के परिवर्तन में 


) √C  
−−
− (A − B ∧ ) √C , 
∞ ∞
∧m = ∧m − (A − B ∧m ∧m = ∧m
∞ ∞
m

A and B are called A तथा B कहलाते है -


(A) van der Waal’s constant (A) वान्डर वाल्स स्थिरांक
(B) Debye-Huckel constants (B) डीबाई-हकल स्थिरांक
(C) Onsager constants
(C) ऑनसेगर स्थिरांक
(D) Critical constants
(D) क्रांतिक स्थिरांक
199. For a solution of two liquids A and B, it was 199. दो द्रवो A तथा B के एक विलयन के लिये यह प्रमाणित
proved that P = X (P − P ) + P . The
A

A

B

B किया गया है की P = X (P − P ) + P  विलयन है
A
∘ ∘ ∘
A B B

solution is :-
(A) आदर्श 
(A) Ideal
(B) अनादर्श
(B) Non - Ideal
(C) अर्द्ध आदर्श
(C) Semiideal
(D) None of the above (D) उपरोक्त में से कोई नही

200. If the solubility of AgCl (formula mass = 200. यदि AgCl (सूत्र भार = 143) की जल में विलयेता
143) in water at 25ºC is 1.43 × 10−4 25ºC पर 1.43 × 10−4 ग्राम प्रति 100 mL है, तो
g/100 mL of solution then the value of Ksp Ksp का मान होगाः -
will be -
(A) 1 × 10−5
(A) 1 × 10−5
(B) 2 × 10−5
(B) 2 × 10−5
(C) 1 × 10−10
(C) 1 × 10−10
(D) 2 × 10−10
(D) 2 × 10−10
Rough-Work
Rough-Work
Rough-Work

You might also like